Surgical Anatomy Flashcards

1
Q

Common shoulder conditions

A

Dislocation/instability
Impingement - irritated inflamed rotator cuff muscle tendons as they pass through subacromial space beneath the acromion resulting in pain, weakness and loss of movement.
Torn rotator cuff
Osteoarthritis

How well did you know this?
1
Not at all
2
3
4
5
Perfectly
2
Q

Anatomy of the shoulder

A
Sternoclavicular joint
Clavicle
Acromioclavicular joint
Acromion
Subacromial bursa
Supraspinatus tendon
Glenohumoral joint
Glenoid labrum - fibrocartilage around scapula cavity for head of humerus to sit
Subscapularis tendon 
Subscapularis muscle
How well did you know this?
1
Not at all
2
3
4
5
Perfectly
3
Q

Anatomy of rotator cuff

A
Subscapularis 
Long head of bicep
Supraspinatus
Infraspinatus
Teres minor
How well did you know this?
1
Not at all
2
3
4
5
Perfectly
4
Q

Age and diagnosis of shoulder

A

10-30yrs most likely instability (more common in athletes, or people with collagen diseases with unstable ligaments)
40-60 more likely impingement syndrome
60-80 likely full thickness cuff tears or osteoarthritis as collagen generation ability declines

How well did you know this?
1
Not at all
2
3
4
5
Perfectly
5
Q

Shoulder stability and the glenoid labrum

A

Fibrocartilage attached around margin of glenoid cavity in scapula for head of humerus to sit in
Increases surface area
Increases depth by up to 50%
Acts as an attachment for ligaments and reduced shock
Resection reduces resistance to translation by 20%

How well did you know this?
1
Not at all
2
3
4
5
Perfectly
6
Q

Dynamic factors for shoulder stability

A

More important than static factors - i.e. glenoid labrum
Joint compression
60% compressive load is needed to dislocate shoulder
Can allow voluntary instability

How well did you know this?
1
Not at all
2
3
4
5
Perfectly
7
Q

Pathology of dislocation

A

External rotation twist causes shoulder to spin out, posterior cuff muscles contract and humeral head rips ligaments in from and tears glenoid labrum making more likely to repeat injury as muscles are now slackened if not repaired
Bankart lesion - damage to glenoid labrum due to anterior shoulder dislocation

How well did you know this?
1
Not at all
2
3
4
5
Perfectly
8
Q

Define arthroscopy

A

Key hole surgery
Minimally invasive surgical technique used on joints

Several basic portals of entry:
Initial posterior
Secondary anterior
Subacromial space space via further posterolateral and lateral portals
Further portals for specific tasks in more complex procedures

How well did you know this?
1
Not at all
2
3
4
5
Perfectly
9
Q

Describe impingement syndrome

A

Rubbing of rotator cuff tendons on acromion process - with age surface becomes irregular instead of smooth
Stage 1 reversible oedema
Stage 2 fibrosis and tendinitis leading to further swelling and further fibrotic process
Stage 3 bone spurs and tendons ruptures and frays
Test for syndrome- Hawkins tests: patient adducts shoulder, internally rotates bringing greater tuberosity under the acromion process - if this is painful it’s likely impingement.
Treatment- injection technique into subacromial bursa in subacromial space or arthroscopic subacromial decompression (bone removal) to flatten and smooth the bone to reduce tendon fraying.
Labrum and tendons stitched together again

How well did you know this?
1
Not at all
2
3
4
5
Perfectly
10
Q

Rotator cuff tears

A

Can be asymptomatic in 15-23% cases as the whole shoulder isn’t needed to function, perhaps only limiting lifting arms above head which in elderly people not common anyway so it goes unnoticed. Deltoid compensates even in supraspinatus etc. Is torn. In ages younger than 59 is 4-13% but above 60 is 20-51%.
Only sudden or gradually larger tears are symptomatic.
25-50% asymptomatic tears become symptomatic in two years
Symptoms relate to tear progression.

How well did you know this?
1
Not at all
2
3
4
5
Perfectly
11
Q

Rotator cuff tear treatment

A

Non operative:
Analgesia - pain relief
Physiotherapy
Activity lifestyle changes
Injection of pain relief eg steroid (long term questionable), anaesthetic and cortisone etc.
Surgical:
Stitch torn cuff muscles together for healing onto greater tuberosity
Zig zag row of tendons, latch down.
No adducting for 2 weeks
If the whole shoulder needs to be opened rather than keyhole surgery patients don’t often recover well
Post op rehabilitation - 3-4 weeks in sling, active movements and physio from then on, resistive movements from 12 weeks ie build strength

How well did you know this?
1
Not at all
2
3
4
5
Perfectly
12
Q

Osteoarthritis

A

Degenerative disease of joints most commonly synovial leading to loss of cartilage and bone changes
Phase 1- Chondrocyte injury
Phase 2- early OA, chondrocyte proliferates and secretes inflammatory mediators, collagen, proteoglycans and proteases. All act together to remodel cartilaginous matrix and initiate secondary inflammatory changes
Phase 3- late OA, repetitive injury and chronic inflammation lead to chronic inflammation and chondrocyte drop out, marked loss of cartilage and extensive subchondral bone changes such as eburnation (very smooth), subchondral cysts from bone fractures or osteophytes.
Primary - occurs in elderly more common in women begins at the 4th decade of life and function declines
Begins usually as wear and tear with repeated minor trauma, hereditary Factors, obesity, ageing etc. Contribute to degeneration.
Secondary - may appear at any age is the result of previous wear and tear like a fracture, inflammation and dislocation beginning the degeneration.

How well did you know this?
1
Not at all
2
3
4
5
Perfectly
13
Q

Osteoarthritis and surgery

A

Can resurface head of joint or replace the entire joint removing pain stimulus
Osteotomy - removal of bone to allow realignment
Arthroplasty - replacement
Arthrodesis - surgical immobilisation by bone fusion

How well did you know this?
1
Not at all
2
3
4
5
Perfectly
14
Q

Surgical approaches to shoulder

A

Aims to access the shoulder joint capsule without damage to nerve of major vessels
3 principal routes:
Deltopectoral- for trauma eg broken humerus, arthroplasty (joint replacement in arthritis), long head of bicep rupture and sepsis
Anterolateral- cuff repair access to subacromial space for tendon attachments, long head of bicep ruptured, acromioclavicular joint decompression, subacromial joint decompression
Posterior - fractures, dislocations, glenoid injury, loose body, sepsis, scapular neck fracture.

How well did you know this?
1
Not at all
2
3
4
5
Perfectly
15
Q

Deltopectoral approach in surgery

A

Access shoulder from the anterior
Cut through skin and fat into the deltopectoral groove all the way to the choroid process
Very constant anatomy here
Remain lateral to tendons as vessels lie medial to this
Into shoulder capsule through subscapularis tendon and expose surface of head of humerus
Branches of circumflex artery at risk here

How well did you know this?
1
Not at all
2
3
4
5
Perfectly
16
Q

Anterolateral approach to shoulder surgery

A

Follow intramuscular plane
Split deltoid muscle
Expose long head of biceps in bicipital groove
Risk of injury to the auxiliary nerve as this wraps around the head of the humerus however only the anterior deltoid would be affected so this poses less risk than posterior entry
Relatively easy approach but deltopectoral used more commonly due to risk of nerve damage in this technique

How well did you know this?
1
Not at all
2
3
4
5
Perfectly
17
Q

Posterior approach to shoulder surgery

A
Intervenous plane followed
Not very common approach due to many nerves and vessels present at risk such as auxiliary which would paralyse entire deltoid and musculocutaneous nerve
Detach deltoid
Expose posterior joint capsule
Reflect infraspinatus
How well did you know this?
1
Not at all
2
3
4
5
Perfectly
18
Q

Broken humerus case study

A

Bad break in upper shaft of humerus treated with rod and screws spanning whole bone
Recurrent falls and loss of proximal hold from the rod results in acromial impingement from compression onto shoulder joint
Remove metalwork and replace with bridging plate, more stable
If still fails then bone grafts and compression plate used to repair damage

How well did you know this?
1
Not at all
2
3
4
5
Perfectly
19
Q

Brachial plexus roots, trunks, divisions, cords and nerves

A

C5 and C6 roots join and branch dorsal scapula nerve and contribute to long thoracic - superior trunk branches subclavian and suprascapula nerves- anterior superior division branch posterior superior nerve to join posterior middle division - lateral cord branches lateral pectoral nerve - musculocutaneous nerve terminates branch contributes to median nerve with C8 and T1
C7 root contributes to long thoracic nerve - middle trunk no branches - posterior middle division sends anterior middle branch to join anterior superior division and joined by posterior superior and inferior from superior and inferior divisions respectively - posterior cord branches upper subscapular, thoracodorsal and lower subscapular nerves - radial nerve branches auxiliary
C8 and T1 join - inferior trunk - anterior inferior division branches posterior inferior branch to posterior middle division- medial cord branches medial brachial cutaneous, medial pectoral and medial anterior brachial cutaneous - branch contribute to median nerve with C5 and 6 and terminates in ulnar nerve

How well did you know this?
1
Not at all
2
3
4
5
Perfectly
20
Q

Musculocutaneous nerve

A

Roots c5-7 supplies three muscles; coracobrachialis, biceps brachii and brachialis
Terminates in lateral cutaneous nerve to forearm
Upper plexus lesion results in low of elbow flexion and lateral forearm numbness can be caused by shoulder dislocations or anterior shoulder surgery

How well did you know this?
1
Not at all
2
3
4
5
Perfectly
21
Q

Radial nerve

A

Roots c5-t1 is the posterior cord from posterior divisions of all three trunks
Lies posterior to auxiliary artery in Axilla
Passes posteriorly via triangular interval with profunda brachi artery
Supplies triceps brachi, anconeus and brachioradialis above elbow is a forearm extensor below elbow supplying extensor carpi radialis, and EXR brevis, extensor carpi ulnaris, extensor digiti minimus, extensor digitorum, extensor indicis, abductor pollicis longus, extensor pollicis brevis, extensor pollicis longus
Sensory effects in wrist capsule through first webspace

How well did you know this?
1
Not at all
2
3
4
5
Perfectly
22
Q

Median nerve

A

Roots c6-t1 branches from medial and lateral cords
No branches in upper arm
Crosses brachial artery lateral to medial to medial boarder or biceps
Enters antecubital fossa medial to brachial artery and biceps tendon
Supplies palmaris longus, flexor carpi radialis, pronator teres, flexor digitorum superficialis and profundus I and II, flexor pollicis longus, pronator quadratus, abductor pollicis brevis, flexor pollicis brevis, opponens pollicis
Passes between two heads of pronator teres and travels through carpal tunnel giving recurrent motor branches to thenar eminence supplying LOAF muscles
Gives off palmar cutaneous branch 4 Cm before wrist

How well did you know this?
1
Not at all
2
3
4
5
Perfectly
23
Q

Ulnar nerve

A

Roots c8 and t1 from medial cord terminal branch
Stays medial in upper arm passes posterior to medial epicondyle within cubical tunnel
Enters medial forearm supplying flexor carpi ulnaris and ulnar half of flexor digitorum profundus
Pierces two heads of FCU travels deep to Flexor digitorum superficialis next to ulnar giving off palmar cutaneous branch
Enters hand via guyons canal and divides into deep motor and superficial sensory branches palmar cutaneous supplying medial palm.
Deep motor branch supplies intrinsic of hand except LOAF muscles (supplies by median)
Dorsal cutaneous branch 5cm proximal to wrist supplies dorsal hand provides digital sensation to the ulnar side for half the digits

How well did you know this?
1
Not at all
2
3
4
5
Perfectly
24
Q

LOAF muscles

A

Lateral two lumbricals
Opponens pollicis
Abductor pollicis brevis
Flexor pollicis brevis

All supplied by median nerve branches

How well did you know this?
1
Not at all
2
3
4
5
Perfectly
25
Q

Brachial plexus lesions: erb-duchenne palsy

A

Inability to move forearm
Affects 1-2 in 1000 births
Caused by undue separation of head from shoulder (shoulder dystocia)
Risks - forceps delivery, vacuum extractor delivery, breach delivery, cephalic presentation with High birth weight above 4kg, prolonged labour and multiparity (twins etc)
Adult risks- fall onto shoulder or sudden axial load, dislocation, tumour (neuroma) and cervical rib arising from C7

How well did you know this?
1
Not at all
2
3
4
5
Perfectly
26
Q

Erbs palsy congenital

A

C5/6 or c7 born without (73-86% cases)
If c7 is involved causes loss of wrist and finger flexion as well as elbow causing true waiters tip
If c4 missing then diaphragm also involved
Just missing c5/6 can recover in three months if c7 involved there is 65% rate full recovery
C5-6 usually combination of musculocutaneous/axillary and median nerve involvement.
Musculocutaneous - loss of elbow flexion
Axillary and suprascapular - loss of shoulder abduction
Unopposed upper and lower subscapular nerve function causes internal shoulder rotation
Sensory loss of lateral upper arm
Waiters hand tip, abducted shoulder, internally rotated arm, extended elbow

How well did you know this?
1
Not at all
2
3
4
5
Perfectly
27
Q

Erbs palsy management

A

Initial support- limb immobilisation and splintage preventing further traction
Good prognosis mostly resolved in 9 months
Physio assisted
Stretch muscle groups preventing contracture
Positional splinting
Kinesiotaping - tape to treat pain by creating space between muscle and dermis layers
Electrical stimulation

How well did you know this?
1
Not at all
2
3
4
5
Perfectly
28
Q

Upper plexus lesions surgery - oberlin transfer

A

Fascicles of ulnar nerve sutured end to end to biceps branch of musculocutaneous nerve (non functional nerve)
Sacrifice FCU and FDS common more often FCU
to give function of elbow flexion from MC nerve
Usually motorcycle injury severing MC nerve
Successfully used in adults and infants obstetric palsy refractory to conservative treatment

How well did you know this?
1
Not at all
2
3
4
5
Perfectly
29
Q

Lower plexus lesion - klumpkes palsy

A

Less common- 1% of obstetric plexus lesions in total
More common in adult trauma eg motorcyclists
Caused by separation of arm from trunk
Roots c8-t1 with or without c7 are injured
Isolated lower root injury is least common
Loss of forearm supination, wrist and fingers are hyperextended
Good elbow and shoulder function
Claw hand - flexion of proximal interphalangeal and extension of metacarpalphalangeal joints
Horners syndrome - contracted pupil, dropping eyelid, local inability to sweat on one side of face due to sympathetic nerve damage in neck on that side.
Recovery is poor especially with horners syndrome below 50%

How well did you know this?
1
Not at all
2
3
4
5
Perfectly
30
Q

Peripheral nerve lesions of upper extremity - cubital tunnel syndrome

A

Ulnar nerve compression at elbow
Cubital tunnel bound by medial epicondyle and medial intermuscular septum anteriorly
Osborn’s fascia superficially
Compression from FDU distally and arcade of struthers (elbow ligament) proximally
Surgically releasing should correct

How well did you know this?
1
Not at all
2
3
4
5
Perfectly
31
Q

Carpal tunnel syndrome

A

Compression of median nerve without carpal tunnel
Occurs in pregnancy, hypothyroidism, rheumatoid arthritis, diabetes any condition with increased oedema
Causes tingling, pain, numb and weakness of wrist and hand
Treated by splinting, physio, injections and surgery to decompress nerve
Surgery- local anaesthetic incision in line with radial boarder of ring finger
Avoids motor recurrent branch of median nerve injury

How well did you know this?
1
Not at all
2
3
4
5
Perfectly
32
Q

Structures in the carpal tunnel

A

9 tendons and the median nerve surrounded by synovium - any condition involving synovial inflammation can cause carpal tunnel

How well did you know this?
1
Not at all
2
3
4
5
Perfectly
33
Q

Guyons canal and nerve compression

A

Ulnar nerve passes through bound by pisiform and hook of hamate
Nerve compressed in different zones gives different symptoms
Zone 1- just past the pisiform causes sensorimotor deficit symptoms
Zone 2- just before hook of hamate causes motor weakness, weak grip but still have sensation
Zone 3- sensory branch past hook of hamate causes sensory disturbance can be caused by H of hamate fracture, tumour, aneurism of ulnar nerve etc.
Treatment- surgery to decompress nerve or correct fracture etc. Causing compression

How well did you know this?
1
Not at all
2
3
4
5
Perfectly
34
Q

Dupuytrens disease

A

Nodular fibroproliferative disorder
Increases with age
Associated with trauma, certain drugs and liver disease
Not a flexor tendon disease but a sudden thickening of fascia and fibrosis causing a curled hand
Advanced stages can have growth of any fibres in the hand
Cords in the fingers can then displace nerves centrally creating disturbance in function further
Treatment- open fasciectomy, cut linearly directly over Palmer disease, expose neurovascular bundles and superficial and fascia attached disease. Excise cord and extend joints protecting nerve and artery. Stitch skin using zig zag incisions to lengthen skin present(Z-plasty).

How well did you know this?
1
Not at all
2
3
4
5
Perfectly
35
Q

Osteoarthritis of Carponetacarpal joint

A

Incidence increases with age associated with general OA
Higher incidence in manual workers
Pain at base of thumb, thenar weakness and wasting and z-deformity of thumb.
Ligaments stretched and allows joint to slide more and more causing hyper extension of MCP joint.
Treatment-
Non steroidal anti inflammatory drugs (NSAID)
Steroids/ hyaluronidase injections
Splintage to maintain muscle mass
Surgery- 70% cases avoid surgery for three years,
joint conserving,
replacement arthroplasty- reconstruction etc.
arthrodesis - fusion of bones
or excisional arthroplasty - trapiziectomy and reconstruction
Weilby technique

How well did you know this?
1
Not at all
2
3
4
5
Perfectly
36
Q

Describe the weilby technique

A

Used to correct osteoarthritis of the CMC joint with Z-deformity
Curved incision over CMC joint extending to wrist
Excise trapezium using osteotome pieces
Incise FCR tendon 50% of width and strip distally
Pass into trapezium cavity and wrap around abductor pollicis longus tendon
Wrap FCR and APL together and stitch remaining tendon into cavity
Free tendon holds tendons in place by trapezium and removes grinding and therefore pain, helps retain strength and corrects deformity
Z-deformity should be corrected and pain relieved

How well did you know this?
1
Not at all
2
3
4
5
Perfectly
37
Q

Colles fracture

A

Fracture 2-3cm proximal to the wrist joint in osteoporotic bone from falling on outstretched hands most commonly
Dinner fork deformity
Dorsal angulation and displacement, radial translation, supination, proximal impaction.
If severe can result in carpal tunnel syndrome due to deformity or haematoma
Needs emergency correction and decompression of tunnel
Treatment-
Manipulate under local anaesthetic and pull to correct level
Longitudinal traction to hold in place but is difficult in younger patients due to movement increasing deformity and reducing distal fragment causing flexion and ulnar deviation
Apply cast- from elbow to MCP joints leaving free fingers and thumb
Check positioning every 7-10 days radiologically
Remove plaster after 6 weeks and start mobilisation follow up physio.
If unsuccessful needs surgery- open reduction and internal fixation some cases may be suitable for wires
Longitudinal incision over PL tendon
Median nerve deep to this/adjacent radial artery is radial to tendon. Use retractors to hold all structures aside. Fracture of radius is beneath FPL and pronator quadratus 5cm distal of volar surface of radius. Plate follows natural curve of radius
Very successful function after
Complications- 3% people, complex regional pain syndrome caused by disturbance of sensory and autonomic supply and blood vessels. Median nerve damage. Rupture of EPL tendon(easy to hit as wraps around head of radius) stiffness, weakness and ulnar impaction.

How well did you know this?
1
Not at all
2
3
4
5
Perfectly
38
Q

Radio carpal joint normal indices

A

Ulnar naturally varies in length compared to the radius but usually equal give or take 2mm either side
Radial height is the distance from the tip of the radial styloid to the articular surface and is approx 11mm
Radial tilt is the angle of tangent across the articular surface compared to perpendicular
Loss of height of change in ulnar variance or loss of volar radial tilt is associated with poor grip strength and wrist function after a fracture - surgery aims to correct this

How well did you know this?
1
Not at all
2
3
4
5
Perfectly
39
Q

Smiths fracture

A

Reverse colles fracture
Falling on to flexed wrists
Distal radial fragment displaced volarly
Extremely unstable causing pain and flexion deformity of wrist
No cast possible as won’t correct properly

How well did you know this?
1
Not at all
2
3
4
5
Perfectly
40
Q

Barton’s fracture

A

An intra-articular fracture of distal radius with dislocation of Radiocarpal joint
Two types - dorsal and volar the latter being more common
Caused by fall on an extended and pronated wrist increasing carpal compression force on dorsal rim
Carpal displacement distinguishes fracture from a smiths or colles fracture
High risk patients have osteoarthritis and poor wrist function

How well did you know this?
1
Not at all
2
3
4
5
Perfectly
41
Q

Management of Barton’s and smiths fracture

A

Distinguish from colles
Spider-Man cast possible but very difficult - wrist supinated and fully extended
Usually needs open reduction - incision and correction of bone - and internal/external fixation

How well did you know this?
1
Not at all
2
3
4
5
Perfectly
42
Q

Open and closed reduction

A

Open - incision into fracture site and correction of bone position
Closed - correction of bone fracture without incision

How well did you know this?
1
Not at all
2
3
4
5
Perfectly
43
Q

Scaphoid fractures

A

Most commonly injured carpal bone
Typically from fall into outstretched hands
Diagnosed in x-ray but very hard to see so if suspicious use CT scan or MRI is most effective
Mostly heal with cast with thumb held but any displacement is best treated with screw fixation
Non union can be very serious causing persistent pain
Percutaneous fixation - cannulated screw to hold scaphoid in place
Vascularised bone graft - radial incision over scaphoid distal radius and scaphoid exposed. EPL, EPB and APL retracted
Window in distal radius made and cube of bone on soft tissue pedicle reflected forward
Gap made in scaphoid filled with bone graft and bone heals with blood supply from distal radius
Immobilise for 8-12 weeks

How well did you know this?
1
Not at all
2
3
4
5
Perfectly
44
Q

Salvage surgery for scapholunate advanced collapse SNAC

A
incision over Carpus 
Extensor tendons retracted
Dorsal capsule incised 
Carpal bones identified 
Scaphoid excised and lunate/capitate/hamate/triquetral fused together with screws after bone prep/burring form rough edge
How well did you know this?
1
Not at all
2
3
4
5
Perfectly
45
Q

Subluxation of proximal interphalangeal joint

A

A partial dislocation
Once Dislocated becomes unusable and prone to further recurrent dislocations
Very difficult to operate as fingers favour fibrosis scarring and so stiffen when incised.
Operative technique hand supine with tourniquet
Transverse wire with centre of rotation in first phalange at PIPJ and parallel wire in second phalange
Bend wires 90degrees bear skin and attach longitudinal wires and slide spring mount over long wires and adjust accordingly
Left with H shape around finger
Steady improvement in range of post operative flexion in patients

How well did you know this?
1
Not at all
2
3
4
5
Perfectly
46
Q

Muscles important in hip surgery

A

Muscles attaching to the ASIS and the deep rotator muscles of the hip - obturator externus, internus, piriformis, gluteus minimus, superior gemellus, inferior gemellus, quadratus femoris.
Muscles attaching to the lesser trochanter aren’t as important more for pelvic surgery access than hip

How well did you know this?
1
Not at all
2
3
4
5
Perfectly
47
Q

Define extensile/non-extensile in terms of surgery

A

Extensile - very long continuation of an internervous plane (between the blood vessels and the nerves) so potentially could bring incision all way down limb etc.
Non-extensile - internervous plane is interrupted by nerve or vessel

How well did you know this?
1
Not at all
2
3
4
5
Perfectly
48
Q

Types of hip replacement

A

Hemi-arthroplasty - no replacement of acetabulum cup

Total arthroplasty includes cup replacing acetabulum

How well did you know this?
1
Not at all
2
3
4
5
Perfectly
49
Q

Approaches to hip replacement

A

Anterior - Smith Peterson
Anterolateral/lateral - Watson Jones, hardinge, charnley
Medial - ludloff
Posterior - Moore

How well did you know this?
1
Not at all
2
3
4
5
Perfectly
50
Q

Anterior approach to hip surgery

A

Used for DDH developmental dysplasia of hip
Synovial biopsies or washout of septic hip
Arthrodesis - surgical immobilisation of bone
Tumour excision
Trapdoor procedure for AVN avascular necrosis
Arthroplasty - hip replacement
Procedure - incision from anterior half iliac crest to the ASIS, curve incision down to run vertically for 8-10cm towards lateral 1/3 of patella. Lateral cutaneous nerve of thigh at risk - causes meralgia paresthetica loss of sensation in anterior thigh.
Follow internervous plane between sartorius (femoral nerve) and tensor fascia latae (superior gluteal nerve) superficially then deep follow between rectus femoris (femoral nerve) and gluteus medius (superior gluteal nerve)
Externally rotates leg to stretch sartorius and accentuate gap between this and tensor fascia lata
Divide fascia 2.5-7cm below ASIS avoiding lateral cutaneous nerve of thigh cut medial to tensor fascia lata and lateral to sartorius and retract muscles. Tie off lateral ascending branch of lateral cutaneous femoral nerve under tensor fascia lata.
Identify and detach two heads of rectus femoris
Strip gluteus medius and minimus from ilium surface and expose capsule of hip joint.
Shave off bone to expose little subchondral bone to fit implant In. Not cemented - biological implant

How well did you know this?
1
Not at all
2
3
4
5
Perfectly
51
Q

Smith Peterson anterior approach to hip surgery strengths and weaknesses

A

Good-
Good for paediatric surgery, protects blood supply
Minimally invasive
Can be extended for pelvic or acetabular repair
Good view of acetabulum
Bad-
Compromised view compared to other approaches
Not most commonly used
Can create bleeding
In field with key nerves eg femoral
Reputation for dislocation post operation

How well did you know this?
1
Not at all
2
3
4
5
Perfectly
52
Q

Difference between hemi arthroplasty and total arthroplasty of hip

A

Hemi - half, only head of femur replaced and fitted back into natural acetabulum, used in cases of neck of femur fracture, fragility fractures etc. (Fall From standing height or less - bones must be fragile to sustain this type of fracture)
Does eventually wear away acetabulum so use standard Exeter stem replacement so that they can fit an Exeter cup for acetabulum in when needed
Total arthroplasty - both acetabulum and head replaced at once

How well did you know this?
1
Not at all
2
3
4
5
Perfectly
53
Q

Anterolateral approach to hip surgery - Watson and sir John Charnley

A

Used for total joint replacement most common method
Open reduction and internal fixation of femoral neck fractures
Synovial biopsies of hip
Biopsies of femoral neck
Incision 2.5cm behind ASIS to tip of greater trochanter and along shaft of femur
No true internervous plane - glut med and tensor fascia lata have common supply - superior gluteal nerve
Incise fascia lata entering trochanteric bursa underneath it.
Watson Jones technique - anteriolateral, expose interval between gut med and TFL generally less destructive method of exposing capsule
Hardinge - direct lateral technique goes transgluteal
Charnley - trochanteric osteotomy removing trochanter - go underneath glut med and remove femur neck and head completely and reflect superiorly to expose capsule or go straight through gluteus medius.

How well did you know this?
1
Not at all
2
3
4
5
Perfectly
54
Q

Dangers of anterolateral approach to hip surgery

A

Ascending branch of left femoral cutaneous artery and veins pass deep to rectus femoris, TFL and gluteus medius all very close to muscles being incised but this isn’t often a problem
Inferior branch of superior gluteal nerve passes 4.5cm above trochanter in gluteus medius can cause excessive traction or injury leading to trendelenburg gait (weaker side of pelvis elevated causing better side to sag often a problem)
Sciatic nerve must be protected whilst dislocating head
Femoral nerve and vascular bundle are close by.

How well did you know this?
1
Not at all
2
3
4
5
Perfectly
55
Q

Strengths and weaknesses of anterolateral approach to hip

A

Charnley approach - easy to dislocate, excellent acetabular exposure, better alignment of hip components, extensile however lots of blood loss, long operative time, difficult to reattach trochanter and possibility of non union of bones, wire breakage and trochanteric bursitis (bursa inflammation causing pain) and cut muscle can scar causing adductor problems
Hardinge approach - usually compared to posterior approach has lower dislocation rate, sciatic nerve injury rate and preserves posterior tissue, extensile. However bad points are superior gluteal nerve injury, trendelenburg gait, limited acetabular exposure, abductor reattachment concern, can’t adjust trochanteric tension and not easy to lengthen and possibility of heterotopic ossification (bone growth in abnormal place)
Watson Jones - uncommon. Need heavy retraction to expose and can damage femoral artery, nerve and veins, very difficult exposure in musculature or obese patients and difficult to access femur. Extensile. Not so ideal for use in hip arthroplasty due to other techniques working better

How well did you know this?
1
Not at all
2
3
4
5
Perfectly
56
Q

Posterior approach to hip surgery

A

Most common for hip arthroplasty
Historically higher dislocation rate but now no longer should be the case if done well
Incision 10-15 Cm curved centred on posterior aspect of greater trochanter
Start 6cm above and posterior to the posterior aspect of greater trochanter
Curve across buttock and continue down shaft of femur
Incise fascia lata expose vastus lateralus
Split fibres of glut max in line with fibres
Retract glut med and min to reveal fat over short external rotators of hip
Identify muscles - piriformis most superficial and behind this is sciatic nerve in front of other rotators, retracting whole mass and sciatic nerve protects nerve
Box or t capsulotomy (flex hip 40 degrees to relax anterior capsule and iliofemoral ligament)
Dislocate hip
Cut 1cm back in tendons to protect ascending branch of medial circumflex artery - supply to femoral head!

How well did you know this?
1
Not at all
2
3
4
5
Perfectly
57
Q

Dangers of posterior approach to hip surgery

A

Sciatic nerve damage
Inferior gluteal artery and nerve damage - within 5 Cm to greater trochanter
Cruciate anastomoses at lower boarder of quad femoris is anastomoses of ascending branch of 1st perforator, descending branch of inferior Epigastric, medial and lateral femoral circumflex arteries

How well did you know this?
1
Not at all
2
3
4
5
Perfectly
58
Q

Strengths and weaknesses of posterior approach to hip

A

Good -
Avoid cutting abductors
Avoid complications of trochanteric osteotomy
Less heterotopic ossification (bone formation at abnormal anatomical site)
Easy exposure and faster rehabilitation
Extensile
Bad-
Increased risk of sciatic nerve injury causing foot drop
Increased risk of posterior dislocation
Increased infection risk

How well did you know this?
1
Not at all
2
3
4
5
Perfectly
59
Q

Medial approach ludloff to hip arthroplasty

A

Very uncommon aside from in paediatrics
Lower limb in frog position
Incision in skin, adductor longus for 6cm then incise aponeurosis
Internervous plane between all adductors and aponeurosis released by blunt dissection
Access lesser trochanter, psoas tendon and inferior femoral neck and head, anterior acetabulum
Minimally invasive
Anterior division of obturator nerve lies on obturator externus down medial thigh between adductor longus and brevis and gracilis
Posterior division of obturator nerve lies in obturator externus and down on adductor Magnus under brevis supplies obturator externus and adductor Magnus
Medial femoral circumflex artery passes round medial side of distal part of psoas - supplies entire femoral head
Good for DDH cases, good before walking age when no bony correction likely needed and some arthrogram advantages (imaging)
Weaknesses - all nerve and blood supply dangers

How well did you know this?
1
Not at all
2
3
4
5
Perfectly
60
Q

Risks associated with hip arthroplasty

A
Infection - 1% deep 2% superficial
Bleeding
Nerve injury
deep vein thrombosis or pulmonary embolism from manipulation 
Dislocation
Limb length discrepancy
Fracture
Loosening
Revision or repeat surgery
Persistent symptoms or pain
stiffness
Death
Delayed/malunion or non-union of bone
How well did you know this?
1
Not at all
2
3
4
5
Perfectly
61
Q

Describe the knee joint

A

Largest joint in body
Modified hinge joint with flexion and extension and secondary rotational movements
Two types of joint - condylar (ovoid) from tibial femoral articulation and the saddle joint from the patella
Extension muscles - vastus lateralis, medialis, medialis and rectus femoris
Rotation muscles - sartorius, gracilis, semimembranosus
Flexion muscles - hamstrings; biceps femoris, semimembranosus, semitendinosus. Gastrocnemius, gracilis, sartorius and popliteus

How well did you know this?
1
Not at all
2
3
4
5
Perfectly
62
Q

Extensor mechanism injuries

A

Injury to the:
Quadriceps and tendon
Medial and lateral retinaculum
Patella and tendon/ligament
Tibial tubercle
Causes presentation patient can’t straighten leg or straight leg raise
Treatment: repair or reconstruct bones and or ligaments if possible, osteotomy, arthroplasty or arthrodesis
Patella- cannulated screw fix in place wires connecting screws to prevent compression splitting the bone
Ligament damage - very complex!
Extracapsular :
tendon quads tendon for patella
Lateral and medial collateral ligaments
Oblique popliteal ligament
Intracapsular :
Anterior and posterior collateral ligaments (ACL Has anterior medial and posterior lateral bundles, PCL has anterior lateral and posterior medial bundles)
Most surgeons only reconstruct either ACL or PCL due to difficulty of reconstruction

How well did you know this?
1
Not at all
2
3
4
5
Perfectly
63
Q

Anterior cruciate ligament injuries

A

Caused mostly by pivoting sports - skiing, basketball, rugby etc.
MRI used to determine, ligaments generally ruptured from femoral insertion
Choices of graft - hamstrings, patella BTB or allograft
Hamstrings have 10% weakness
Grafts taken from cadavers
Bone grafts can heal better than hamstring acts as fracture heal. Best method is debatable

How well did you know this?
1
Not at all
2
3
4
5
Perfectly
64
Q

Meniscus and injuries

A

Fibrocartilage
Load bearing function
Medial cup C shaped and smaller
Lateral is O shaped and larger
Medial is attached to the knee capsule and is more likely to tear
Lateral is more mobile not attached to capsule so less likely to tear
Very fragile blood supply making meniscal tears very difficult to heal - best healing is a tear within 3mm of the rim as this has the most supply of blood
Bucket handle tear- middle part sits in the middle due to tear on edge and Presents as cant extend knee past certain point - locking.
Can transplant in correct criteria of patient eg young, ACL in tact, good BMI etc.
Resection partially increased knee load by 65% or total increases peak load 235%

How well did you know this?
1
Not at all
2
3
4
5
Perfectly
65
Q

Osteotomy

A

Used for correction of varus (deformity involving displacement of part of limb toward midline)
Valgus (deformity away from midline)
In a stable joint with no inflammation and only pain in movement
In knees if patient ACL has been operated on or damaged then cannot have osteotomy due to dangers too great to perform operation

How well did you know this?
1
Not at all
2
3
4
5
Perfectly
66
Q

Arthroplasty for knees

A

Unicompartmental - only affected section replaced
Takes away less bone and soft tissue than total arthroplasty so should act more like native knee behaves more normally
Controversial
Total replacement leaves only collateral ligaments and epicondyles
If patient has torn ACL cannot have arthroplasty due to massive increase in surgery risk
Procedure -
Anterior approach - Midline incision into quad tendon on medial side of patella through retunaculum go round tendon to medial tuberosity
Lateral approach - Incision between tibia and gastrocnemius natural plane
Posterior - for fractures near the back, problematic as many vessels and nerves here. Once in the knee natural plane is present but many structures in the way.

How well did you know this?
1
Not at all
2
3
4
5
Perfectly
67
Q

Complications of knee surgery

A
Infection
Bleeding
Nerve injury
Soft tissue injury
DVT
stiffness and swelling
Fracture
Dislocation
Revision surgery needed
How well did you know this?
1
Not at all
2
3
4
5
Perfectly
68
Q

Movements of the foot

A
Supination - shift foot toward midline
Pronation - away form midline 
Inversion
Eversion (more ankle movement than pro and sup)
Dorsiflexion
Plantar flexion
How well did you know this?
1
Not at all
2
3
4
5
Perfectly
69
Q

Ankle anatomy

A
Fibula
Tibia
Talus
Two joints 
Three main ligament groups -
Syndesmosis
Deltoid (medial)
Lateral ligament complex
Main movement is dorsi flexion around 20 degrees average and plantar flexion 45 degrees roughly in Sagittal plane
How well did you know this?
1
Not at all
2
3
4
5
Perfectly
70
Q

Talus shape and movement

A
Frustum shape (cone with pointy bit cut off) 
Moves roughly alone sagittal plane in hinge like movements very simple joint 
Very stable in dorsiflexion due to fitting against the tibia more but unstable in plantarflexion due to angle of joint therefore more likely to twist ankle in this position
How well did you know this?
1
Not at all
2
3
4
5
Perfectly
71
Q

Syndesmoses of ankle

A

Only fibrous joint in the limb on the ankle
Formed by interosseous membrane between fibula and tibia
Anterior inferior talofibular ligament from outer anterioinferior boarder of fibula to the neck of talus into distal articular cartilage function to restrain inversion in plantar flexion and resist anterolateral translation of talus in mortise
Posterior inferior talofibular ligament extends from posterior boarder of fibula to posteriolateral tubercle of talus function to help ankle stability (very strong) when lateral ligament complex is intact and is under greatest strain in ankle dorsiflexion and limits posterior talus displacement

How well did you know this?
1
Not at all
2
3
4
5
Perfectly
72
Q

Deltoid ligament of ankle

A

Very well vascularised heals very well and hardly ever needs surgery due to excellent blood supply
Formed of posterior tibiotalar part, Tibionavicular part And tibiocalcaneal part on the medial side of the ankle are very thick and well joined

How well did you know this?
1
Not at all
2
3
4
5
Perfectly
73
Q

Lateral ankle ligaments

A

Thinner and weaker than deltoid on medial side more likely to need surgery
ATFL most likely to rupture in ankle sprains - function to come forward off fibula and stop talus sliding so often gets overstretched and this increases arthritic risk from sliding and increased gap between bones
Achilles
PITFL
PTFL
calcaneofibular ligament
AITFL
ATFL
All much less vascularised than Deltoid do heal Less well

How well did you know this?
1
Not at all
2
3
4
5
Perfectly
74
Q

Why is the Ankle small surface area but transmits loads greater than hip or knee but is less likely to develop arthritis

A

Joint is very congruent fitting together well leaving little movement for bone to rub away cartilage etc.
Strong ligaments hold bones together very well preventing sliding
Very simple joint movements allowed compared to the knee or hip
Very unlikely to ever see primary arthritis only really every secondary arthritis following trauma to the ankle or ligaments etc.

How well did you know this?
1
Not at all
2
3
4
5
Perfectly
75
Q

Muscles of the ankle

A

Plantar flexors:
Gastrocnemius - crosses knee
Soleus
Plantaris - little contribution
Innervated by tibial branch of sciatic nerve mainly s1 and 2
Form common Achilles’ tendon for gastrocnemius and soleus
Dorsiflexors:
Tibialis anterior mainly
Peroneus tertius
Innervated by deep branch of perineal nerve mainly l4 and 5

How well did you know this?
1
Not at all
2
3
4
5
Perfectly
76
Q

Weber classification of ankle fracture

A

Weber A - fracture of fibula below line of syndesmosis - no injury to ligaments so long as bones heal ankle will become stable again, good prognosis
Weber B - fracture at line of syndesmoses most common type 70% of fractures from twisting ankle. Can be stable or unstable making management very difficult and therefore prognosis difficult unsure how best to treat
Weber C - higher fibula fracture above line of syndesmoses however to actually get this fracture force must have been great enough usually to also twist and tear syndesmoses ligaments so likely show complete ankle dislocation. Deltoid ligament also often torn very unstable break almost always needs surgery

How well did you know this?
1
Not at all
2
3
4
5
Perfectly
77
Q

Hind foot and triple joint

A

4 bones : talus, calcaneus, navicular and cuboid
3 joints that act as one (one can’t move without the other two also moving) : subtalar, talonavicular and calcaneocuboid joints
Responsive for inversion and eversion of heel (varus/valgus)
A valgus heel produces a flexible foot good for absorbing shock from impact like jumping etc.
A varus heel helps produce a stiff foot arch good for pushing off creating force for running etc.

How well did you know this?
1
Not at all
2
3
4
5
Perfectly
78
Q

Subtalar joint biomechanics

A

Subtalar joint of heel converts tibial rotation into forefoot supination and pronation and vice versa
This is due to axis of joint roughly 45 degrees to the foot therefore acts like a mitred hinge, twisting the forefoot or tibia causing the other to also rotate
This is useful for shock absorption as angle can help transfer impact to muscles rather than joints from rotation
Subtalar joint very important for shock absorption during gait or accommodating for uneven ground
Is important in energy conservation to make foot solid strut during toe off in gait cycle

How well did you know this?
1
Not at all
2
3
4
5
Perfectly
79
Q

Muscles of hind foot

A

Tibialis posterior most important hind foot inverter swinging heel into varus so that Achilles’ tendon pulls more in line and therefore is more efficient
Innervated by tibial nerve
Broad insertion into midfoot but mainly navicular and medial cuneiform
Passes behind medial malleolus
Peroneus brevis inserts into base of fifth metatarsal and is main Everter of foot
Peroneus longus inserts into base of first metatarsal and plantar flexes it
Both muscles innervated by superficial perineal nerve mainly l5

How well did you know this?
1
Not at all
2
3
4
5
Perfectly
80
Q

Hindfoot antagonistic muscle pairs

A

Tibialis posterior vs peroneus brevis
Tibialis anterior vs peroneus longus
A weak tib posterior leads to a flat valgus foot
Weak tibialis anterior and brevis leads to varus and cavus (very arched foot) eg caused by a demyelination disease etc. Other peripheral neuropathies

How well did you know this?
1
Not at all
2
3
4
5
Perfectly
81
Q

Talus injury’s

A

The talus is covered in cartilage not much soft tissue attachment meaning it’s prone to avascular necrosis due to lack of blood supply and hindfoot arthritis is more likely from dislocations etc:
Blood supply comes from perforating perineal arteries and anterior tibial artery and tarsal canal artery.
Tarsal canal and branch of anterior tibial anastomoses forming the sinus tarsi artery

How well did you know this?
1
Not at all
2
3
4
5
Perfectly
82
Q

Midfoot and lisfranc ligament

A

Navicular, cuboid, three cuneiforms and the tarsometatarsal joints
Key region for surgery - lisfranc ligament and second tarsometatarsal joint.
Lisfranc ligament very small from the medial cuneiform to the base of second TMT
The intermediate cuneiform next to medial recesses in where the base of second TMT sits tightly very strong and stable part of foot
Injury to this area would cause arch collapse and huge instability
Injury to ligament is very common in American football - 0.2% all fractures so not common elsewhere
Diagnosed with MRI
Poor prognosis very painful
Widening of TMTjoints and popping up of 1/2nd metatarsal
Surgery needed- longitudinal incisions on dorsal foot avoiding the dorsal pedal artery and deep fibula nerve

How well did you know this?
1
Not at all
2
3
4
5
Perfectly
83
Q

Other midfoot ligaments

A

Long and short plantar ligaments
Calcaneonavicular (spring) ligament
The head of talus rests on this ligament mesh and often tears this with talus damage

How well did you know this?
1
Not at all
2
3
4
5
Perfectly
84
Q

Forefoot anatomy

A

5 Metatarsalphalangeal joints, 1 interphalangeal joint, four proximal and four distal interphalangeal joints

How well did you know this?
1
Not at all
2
3
4
5
Perfectly
85
Q

Truss and windlass mechanism

A

Raising arch foot height increases foot stability
As arch rises toes are extended
As arch sinks toes are flexed touching floor giving more stability
Load spreading across foot using plantar fascia tightening and stretching from movement

How well did you know this?
1
Not at all
2
3
4
5
Perfectly
86
Q

Hallux valgus

A

Most common angular deformity of toes
Genetic predisposition from apes opposing toes or bad shoes
intermetatarsal angle widens and big toe (hallux) overlaps others
Doesn’t bend efficiently and likely prone to arthritis due to large gaps between bones
Rheumatoid arthritis rapidly advances this condition as it’s a disease that increases all joint instability
Treated with osteotomy - very straight forward to realign hallux appropriately
The earlier the better
Blood supply to the metatarsal head from the plantar artery can be detached accidentally causing avascular necrosis and arthritis which we don’t want

How well did you know this?
1
Not at all
2
3
4
5
Perfectly
87
Q

Forefoot blood and nerve supply

A

3 main arteries - posterior tibial, anterior tibial and peroneal artery formed at the trifurcation of the popliteal artery
Posterior tibial is main supply
Terminal branch of anterior tibial (dorsalis pedis) pierces through first intermetatarsal space to anastomoses with the plantar artery
Nerves -
Saphenous (terminal femoral branch) innervates medial ankle
Perineal, superficial and deep - deep innervates first webspace and superficial had largest area of dorsal foot and lateral ankle
Medial and lateral plantar and calcaneal nerves from tibial nerve - medial largest area of plantar medial foot and most toes, lateral the lateral, calcaneal the heel
Sural nerve - lateral plantar foot

How well did you know this?
1
Not at all
2
3
4
5
Perfectly
88
Q

Muscles of the foot

A

Instrinsic - lumbricals, interossei, short flexors, extensors and muscles of the hallux and fifth toe
Extrinsic - long flexors and extensors, dysfunction of these leads to claw toes etc.
Would rarely operate on plantar side of foot due to excess of muscle and neurovasculature present
Deepest layer- tibial posterior, peroneus longus and interossei
Next- two short flexors, one adductor
Next- two long flexors, lumbricals and quadratus plantae
Superficial layer- two abductors and flexor digitorum brevis

How well did you know this?
1
Not at all
2
3
4
5
Perfectly
89
Q

Baxter’s neuropathy

A

Nerve entrapment syndrome that results in compression of inferior calcaneal nerve

How well did you know this?
1
Not at all
2
3
4
5
Perfectly
90
Q

Toe deformities

A

Mallet toe - stubbing
Claw toe - muscle weakness
Hammer toe
Curly toe - common congenital problem from long flexors tightening

All descriptive of looks

How well did you know this?
1
Not at all
2
3
4
5
Perfectly
91
Q

The reconstructive ladder

A
Free tissue transfer
Local tissue transfer
Tissue expansion
Skin grafts
Delayed primary closure - If wound is infected then they clear the infection before closing the wound
Primary intention
Secondary intention

(Going up)

How well did you know this?
1
Not at all
2
3
4
5
Perfectly
92
Q

Wound healing overview

A

Haemostasis and coagulation
Inflammation
Proliferation and migration
Remodelling

Factors affecting wound healing :
Infection
Blood flow
Steroids
Malnutrition
Diabetes
Tension lines of skin
Radio or chemotherapy
How well did you know this?
1
Not at all
2
3
4
5
Perfectly
93
Q

Main types of skin cancer

A

Squamous cell- can spread to lymph nodes and come back but if excised early small chance of return
Basal cell - doesn’t spread, least worrying. 4mm extra taken out around growth to ensure all is removed and won’t reoccur.
Melanoma - spreads and invades multiple skin levels. 2mm taken either side of tumour or often retake 2-3cm away

How well did you know this?
1
Not at all
2
3
4
5
Perfectly
94
Q

Managing skin cancer

A
Non surgical :
Manage risk factors eg sun cream
Topical treatments like cryo (freezing off) or chemotherapy
Chemo or radiotherapy 
Surgical :
Excision with or without reconstruction
Management of regional lymph node basin
How well did you know this?
1
Not at all
2
3
4
5
Perfectly
95
Q

Types of burn

A

Superficial epidermal - epidermis damaged, skin reddens and swells but no blisters
Superficial dermal - epidermis and part of dermis burn, skin is pale pink and painful, small blisters
Deep dermal/partial thickness - epidermis and dermis damaged, skin red and blotchy, dry or moist and swollen and blistered. Painful or numb
Full thickness burn - all three layers of skin burnt and damaged tissue underneath may be blackened or pale while skin is dry and white/brown/black with no blisters skin may be leathery or waxy.

How well did you know this?
1
Not at all
2
3
4
5
Perfectly
96
Q

Why have heaters in burns units

A

Burns loose the protective barrier of skin so water loss and heat loss is significantly increased so need heaters and hydration constantly to maintain normal homeostasis

How well did you know this?
1
Not at all
2
3
4
5
Perfectly
97
Q

Types of skin graft

A

Full thickness - whole dermis generally used for smaller wounds on highly visible parts of body like the face as unlike split thickness grafts they blend in better with the skin around them and tend to have better cosmetics
Split thickness - removing epidermis and part of dermis moved and placed on recipient site
Graphs can have fenestrations in large graft areas to encourage the skin to stretch and so needing less skin to cover more area

Autograft - using patients own skin
Allograft - donor skin
Xenografts - donors skin from animals usually pigs

How well did you know this?
1
Not at all
2
3
4
5
Perfectly
98
Q

Geometric shapes and reconstruction

A

Geometric shapes especially in facial surgery can give better cosmetics and mean that skin graft isn’t needed from other body parts just use natural laxity of skin to move and stretch to cover exposed area
This can also allow following natural lines on skin eg wrinkles better hiding the scar
Faces have a lot of natural laxity so good to use this technique

How well did you know this?
1
Not at all
2
3
4
5
Perfectly
99
Q

Breast reconstruction tissue sources

A

Breast implant synthetic (not for chemo or radiotherapy patients as implant will shrink and become very painful)
Natural tissue sources-
Latissimus dorsi can be rotated round still attached to blood supply becoming breast tissue. regional flap as original blood supply isn’t removed. Is a sensational flap will have sensation
Insensate flaps- LSGAP from upper buttock or lower buttock
TUG flaps from medial thigh
DIEP, SIEA or free/pedicled TRAM flaps from stomach adipose tissue
Plugging arteries from donor tissue into internal thoracic arteries in Breast allows health tissue to remain.

How well did you know this?
1
Not at all
2
3
4
5
Perfectly
100
Q

Skull anatomy

A

Neurocranium derived from neural crest- protects the brain split into the calvarium (roof) and cranial base (floor).
Sections anterior to posterior: frontal bone with ethmoid bone in middle, sphenoid, temporal, occipital with parietal lateral to last three.
Viscerocranium compromises of (superior to inferior) - nasal bone medial, lacrimal, maxilla, zygomatic most lateral. Inferiorly is the mandible. Inside nose bits superior is palatine, medial the vomer and inferiorly the inferior nasal concha.

How well did you know this?
1
Not at all
2
3
4
5
Perfectly
101
Q

Laryngeal cancers

A

Mostly squamous cell
Can affect supraglottis, glottis or subglottis
Rate of cancer rose in men until early 1990s have fallen steadily since then
Hoarse voice common presentation

How well did you know this?
1
Not at all
2
3
4
5
Perfectly
102
Q

Supraglottic laryngeal carcinoma

A

Approximately 40% of laryngeal cancers
Usually present with dysphasia or metastatic neck node
Late presentation is common
Management - supraglottic laryngectomy and radiotherapy
Total laryngectomy reserved for residual or recurrent disease

How well did you know this?
1
Not at all
2
3
4
5
Perfectly
103
Q

Glottic laryngeal carcinoma

A

60% all laryngeal cancers
Usually presents early with dysphonia
Management - radiotherapy for early rumours 95% cure rate for T1 lesions
Total laryngectomy and neck dissection for residual or recurrent disease

How well did you know this?
1
Not at all
2
3
4
5
Perfectly
104
Q

Orbital rim fractures

A

Fracture of any bones forming the outer rim of bony orbit on face
Usually at suture line as this is weakest point
Seven bones forming: frontal, zygomatic, maxillary, lacrimal, ethmoid, palatine, sphenoid.

How well did you know this?
1
Not at all
2
3
4
5
Perfectly
105
Q

Orbital blowout fractures

A

Partial herniation of orbital contents through one of its walls
Usually caused by blunt trauma
Medial and inferior (floor) walls weakest most commonly floor
Entrapment of inferior rectus causes paralysis of upward gaze must check vision and eye movements rigorously
Won’t be able to look upward in damaged eye

How well did you know this?
1
Not at all
2
3
4
5
Perfectly
106
Q

Le fort classification

A

Floating palate fracture - below the nasal zone is fractured and separated
Floating maxilla- fracture from maxilla to eye sockets
Floating face/craniofacial fracture most severe fracture through maxilofrontal suture line slicing face in half

How well did you know this?
1
Not at all
2
3
4
5
Perfectly
107
Q

Mandibular fractures

A

Mostly left sided fractures as most people are right handed and punch people
Relatively common especially in men
Approximately 60% are bifocal
Present with trismus(jaw spasm), malocclusion(can close mouth properly), chin paraesthesia (disruption to mental branch of inferior alveolar nerve)

How well did you know this?
1
Not at all
2
3
4
5
Perfectly
108
Q

Frontal bone fractures

A

Needs significant force 100-200x greater than gravity
Present with forehead paraesthesia from damaged supraorbital nerve and rhinorrhea (nasal cavity filled with significant amounts of mucus fluid)

How well did you know this?
1
Not at all
2
3
4
5
Perfectly
109
Q

Embryology of Face

A

Development occurs mainly between 4-8 weeks gestation
Lower jaw (mandible) is the first to form at 4 weeks.
4th week- maxillary prominences and mandibular prominences form laterally. End of 4th week nasal placode develops (embryological nose)
Week 5- mesenchymal cells proliferate in lateral sides of nasal placodes to form medial and lateral prominences
Medial will form bridge and septum or nose lateral will form nostrils
Week 6- medial nasal prominences fuse
Week 7- maxillary prominences fuse with nasal prominences to form the palette

How well did you know this?
1
Not at all
2
3
4
5
Perfectly
110
Q

Cleft lip

A

During week 6-7 of gestation maxillary prominence fails to fuse with nasal prominences on both or either side during facial development creating cleft lip.
Unilateral incomplete cleft lip - one side of lip is partially non fused/formed
Unilateral complete - one side of lip is entirely not fused up to nostril
Bilateral complete - both sides of lip not fused up to nostrils
Causes feeding problems and speech problems later on in life so needs to be surgically corrected very early around 10 weeks old. If bilateral may do one side then the other separately.

How well did you know this?
1
Not at all
2
3
4
5
Perfectly
111
Q

Cleft palate

A

During week 6-7 of gestation maxillary prominence fails to fuse with nasal prominences on both or either side during facial development creating cleft palate. Often seen alongside cleft lip
Maxillary processes fail to fuse posteriorly.
Causes feeding and speech problems so needs surgery at around 6-10 months using a temporary palate fitted until surgery to allow feeding.
Unilateral incomplete - only posterior palate involved
Unilateral complete cleft palate and lip - whole palate and one side of lip involved
Bilateral complete cleft lip and palate - whole palate and both sides of lip non-fused and undeveloped

How well did you know this?
1
Not at all
2
3
4
5
Perfectly
112
Q

Sutures of skull

A

Fibrous joints unique to the skull
In adults the main sutures are : coronal (transverse across frontal), Sagittal (from occipital fontanelle to frontal) and lambdoid (transverse posteriorly through occipital fontanelle)
Join at frontal and occipital fontanelles
In children also have the Metopic suture from frontal fontanelle to anterior of frontal bone.
Metopic fuses before one years
Sagittal around 30 years
Coronal and lamboid around 25 years.

How well did you know this?
1
Not at all
2
3
4
5
Perfectly
113
Q

Craniosynostosis

A

Defined as premature fusion of one or more cranial sutures
Incidence between 1 in 2000 and 1 in 5000
Restricts growth perpendicularly to the fused suture
Compensatory growth at other cranial sutures
Trigonocephaly - only in children under 1 as Metopic suture normally fused by then. Frontal bone grows anteriorly but not transversely. Pointy head.
Scaphocephaly - fused Sagittal suture causes boat shaped head. Only anterior-posterior growth occurs no transverse. Try to surgically re-open Sagittal suture.
Plagiocephaly - fused coronal suture. Only one side fused early no growth transversely that side compensated growth on other side. Can be mistaken in children that lie on one side for long time deforms skull similar to this just due to infant skull pliability.
Brachycephaly - both coronal sutures fused shortened anterior-post growth excess transverse growth. Needs early management can be life threatening from airway obstruction/increased ICP needs frontal orbital advancement.

How well did you know this?
1
Not at all
2
3
4
5
Perfectly
114
Q

Embryology and anatomy of the pinna

A

Starts at 6 weeks gestation
First and second Branchial arches
Mesoderm condensed to form 6 hillocks of His
Hillocks fuse at week 12 to form auricle
Fully formed at 20 weeks
Reaches adult size at age 9
Each hillocks forms an anatomical location of the auricle : tragus, helical crus, helix, anti helix, antitragus, lobule.
Concha formed by first branchial grove ectoderm: concha cavum (middle), concha cymba (upper), intertragus incisor(lower)
Nerve supply from auriculotemporal (V3), lesser occipital (C2) and great auricular (C2+3).

How well did you know this?
1
Not at all
2
3
4
5
Perfectly
115
Q

Development of external auditory canal

A

8th week gestation
Thickening of ectoderm in first pharyngeal grove with grows towards middle ear
Concha cavum deepens
At 21 weeks ectodermal core reabsorbs
Competed by 28 weeks
Ossification occurs around age 3 and fully grown by 9

How well did you know this?
1
Not at all
2
3
4
5
Perfectly
116
Q

Embryology of Eustachian tube and middle ear

A

Starts at 3 weeks
Third pharyngeal arch enlarges and compresses the space between second pharyngeal arch and first pouch
Develops into the Eustachian tube and lateral out pouch becomes middle ear space
Pneumastidation of middle ear begins at ten weeks
Tympanic membrane appears at 28 weeks
Formed of three layers : ectoderm squamous layer, mesoderm fibrous layer, endoderm mucosal layer.
Middle ear is well formed at birth

How well did you know this?
1
Not at all
2
3
4
5
Perfectly
117
Q

Muscles of facial expression

A

All share common embryonic origin from second pharyngeal arch all innervated by cranial nerve 7(VII)
Groups of muscles: orbital, nasal and oral
Orbital group - orbicularis oculi muscle is split into outer part of orbit and inner palpebral part. Closes eyelid. Corrugator supercilii muscle is posterior to OO and draws eyebrows together to frown.
Nasal group - nasalis muscle has two parts; transverse and alar. Transverse compresses nares,alar opens them(flares nostrils). Procerus muscle pulls eyebrows downward. Depressor septi nasi muscle pulls nose inferiorly opening up the nares (flare nostrils).
Oral group - orbicularis oris muscle encloses the opening of oral cavity, closes and purses mouth. Buccinator muscle located deep to other facial muscles pulls cheek inward preventing build up of food in oral cavity.

How well did you know this?
1
Not at all
2
3
4
5
Perfectly
118
Q

Facial nerve paralysis upper and lower MN lesions

A

If an upper motor neurone lesion then forehead sparing is seen - wrinkles on forehead remain present and forehead has sensation
If lower MN lesion then entire face will have paralysis on side of lesion.

How well did you know this?
1
Not at all
2
3
4
5
Perfectly
119
Q

Head and neck cancer

A

Complex cancer and anatomy.
Disruption to speech, swallowing and cosmetics.
Types : oral, pharyngeal (oropharyngeal and naopharyngeal), laryngeal cancers
Managed with chemo or radiotherapy, surgery or palliative care depending on type and spread of cancer.

How well did you know this?
1
Not at all
2
3
4
5
Perfectly
120
Q

Oral cancers

A

90% oral cancers are squamous cell carcinomas
50% involved the tongue, tonsils next common
Associated with smoking, HPV and alcohol.
High prevalence in Indian subcontinent due to beetlenut chewing
Treated by surgery and radio/chemotherapy combo. Occurs commonly in glossopharyngeal arch, pharyngopalatine arch, palatine tonsil and tongue most frequently.
If surgery needed can use free flap from fibula along with peroneal artery and vein for blood supply to reconstruct cheek if has spread to mandible.

How well did you know this?
1
Not at all
2
3
4
5
Perfectly
121
Q

Pharyngeal cancers

A

Oropharyngeal - 75% squamous cell carcinoma, tonsils and faucial pillars most common sites, associated with smoking and HPV, usually present with throat pain or globus sensation. Usually treated with radio/chemotherapy. Surgery only in cases of neck node metastasis.
Nasopharyngeal - mostly squamous cell carcinoma associated with EBV/Chinese descent. Highest incidence in Southeast Asia and can invade through foramen ovale causing facial pain by cranial nerve 5. Some have protracted course with overall 5 year survival rate 60-80%

How well did you know this?
1
Not at all
2
3
4
5
Perfectly
122
Q

Maxillary fractures

A

Relatively straight forward unless fracture over infraorbital foramen where nerve facial travels through

How well did you know this?
1
Not at all
2
3
4
5
Perfectly
123
Q

Microtia

A

One in 7000
Failure of hillocks of his to develop and fuse
Can be uni or bilateral (3:1 unilateral) and more common in males in right ear.
Associated with multiple other pathologies. Graded 1-4 4 being nothing formed.
Patients usually wear clip on prosthetics or reconstructed using grafts at 5 years

How well did you know this?
1
Not at all
2
3
4
5
Perfectly
124
Q

Pre auricular sinus

A

Failure of the first and second hillocks to fuse or remnant of first brachial cleft
May become infected and excision needed
Sinus may extend long into ear canal or mandible where damage to facial nerve is possible
Generally large chunk of tissue removed to ensure all is gone

How well did you know this?
1
Not at all
2
3
4
5
Perfectly
125
Q

Bat ears

A

Missing anti helical fold of ear treated with pinnaplasty where ears are pulled back and helix is reformed
Furnas and mustarde sutures used on posterior of ear where skin has most loose adherence and can hide scar
If caught young enough can just use headband to pull ears back naturally.

How well did you know this?
1
Not at all
2
3
4
5
Perfectly
126
Q

External auditory canal

A

Redirects and redistributes sound
Lateral third has thick skin tightly adhered to underlying cartilage, hair and ceruminous glands
Inner two thirds has thin skin, no hair, very bony and very sensitive.
Nerve supply from the auriculotemporal nerve, lesser occipital, facial and vague nerves.

How well did you know this?
1
Not at all
2
3
4
5
Perfectly
127
Q

Eustachian tube anatomy

A

Runs anteromedially from middle ear to nasopharynx grows from 17-35mm in adults to produce an incline of 45 degrees
Medial 2/3 is cartilaginous lined by pseudostratified columnar ciliated epithelium while lateral third is bony and covered with cuboidal epithelium
Opened by tensor veli palatini (V3) and levator veli palatini (X)
Normal middle ear pressure kept at 200-300mmH2. Children often experience negative pressures muffling hearing

How well did you know this?
1
Not at all
2
3
4
5
Perfectly
128
Q

Tympanic membrane

A

Three layers - squamous outer layer continuous with external auditory canal, fibrous layer circular and radiating fibres and mucosal layer respiratory epithelium continuous with middle ear mucosa
Divided into pars tensa and pars flacida
Pars tensa - 80 and 55mmH2 area buckled on sound presentation converting sound to motion energy
The malleus is embedded in the tympanic membrane and energy transmitted to ossicular chain.
When viewing tympanic membrane can see lateral process of malleus, good for orienting as will always point anteriorly

How well did you know this?
1
Not at all
2
3
4
5
Perfectly
129
Q

Glue ear and otitis media

A

Infections of ear can lead to one another
Glue ear is fluid accumulation otitis media is pus and infection
Those with glue ear more commonly get otitis media problems
Otitis media is chronic while glue ear is acute
Surgical management- NICE guidelines for 12years or older, no Down’s syndrome or cleft palate patients qualify due to having different ear anatomy
Patients can have hearing difficulty, speech and language delay, ear infections or otalgia, nasal obstruction, behavioural problems, balance difficulties, tinitus and hyperacusis(very sensitive to sound)
Performed if better ear has hearing of 25-30dBHL or worse at 0.5,1,2 and 4 kHz or earlier if impact on social, educational status deemed significant.
Ventilation tubes inserted. Adenoidectomy maybe but not recommended in presence of frequent URT symptoms. Hearing them reassessed. Otovent balloon auto inflation may be considered during active observation.
Down’s syndrome and cleft palate have narrow ear canals and multiple pathologies so need MDT and hearing aids

How well did you know this?
1
Not at all
2
3
4
5
Perfectly
130
Q

Perforations and grommets

A

Perforations mostly heal on their own in the ear however if there is infection or complications then surgically need to close them.
Inserting a grommet to ventilate the ear drains the fluid and prevents reaccumulation

How well did you know this?
1
Not at all
2
3
4
5
Perfectly
131
Q

Internal acoustic meatus

A

1 Cm long
Runs in Petrous temporal bone
Perforated end plate at lateral end separating IAC from inner ear
Cranial nerves VII and VIII and labyrinthine artery and vein pass through end place to middle ear
Cranial nerve VIII divides into superior and inferior divisions

How well did you know this?
1
Not at all
2
3
4
5
Perfectly
132
Q

Middle ear

A

Located in petrous part of temporal bone
Includes tympanic cavity and and epitymapnic recess
Posterosuperiorly connects with mastoid air cells through the additus
Mucous membrane is continuous with Eustachian tube and mastoid
Contains - ossicles, tendons stapedius and tensor tympani, chorda tympani (CN VII) and tympanic plexus (CN IX)

How well did you know this?
1
Not at all
2
3
4
5
Perfectly
133
Q

Walls of tympanic cavity

A

Superior tegmental wall - formed by tegmen tympani, thin bone on floor of middle cranial fossa separates middle ear from dura.
Inferior jugular wall - underneath is superior bulb of inferior jugular vein rarely this can be high and extend to middle ear or lack bony covering
Lateral membranous wall - mostly tympanic membrane but superiorly epitympanic recess
Labyrinthine medial wall - separates middle ear from inner ear, promontory.
Carotid anterior wall - carotid canal anterior with internal carotid artery running through it.
Mastoid posterior wall - additus is superior part, connects middle ear to mastoid air cells. Fallopian canal containing cranial nerve VII descends between wall and mastoid antrum contains pyramidal eminence.

134
Q

Bones of middle ear

A

Malleus- derived from first branchial arch. Handle and lateral process embedded in tympanic membrane. Head lies in epitympanic recess. Neck found in pars flaccida. Tip forms the umbo, tensor tympani tendon inserts into the handle, chorda tympani crosses medial surface and has four ligaments, anterior, superior, lateral and tensor tympani.
Incus - short and long limb, body lies in epitympanic recess, two ligaments superior and posterior. Articulates with malleus and stapes via synovial joints.
Stapes - head is directed laterally and articulates with lenticular process of incus. Transmits sound to oval window via syndesmosic joint. Stapedial tendon runs from apex of pyramidal process to neck.

135
Q

Muscles of middle ear

A

Tensor tympani arises from cartilaginous ET greater wing of sphenoid and petrous part of temporal bone
CN VIII innervates
Pulls handle of malleus medially designed to protect inner ear by tightening when loud sounds heard preventing damage but consistent loud noise prevents protection
Strapedius muscle located inside pyramidal eminence, tendon inserts into neck of stapes
Innervated by CN VII. Pulls stapes posteriorly reducing range and excessive movement.

136
Q

Retraction pockets and cholesteatoma

A

Pockets filled with cholesteatoma have constant discharge and infection leading to bone erosion need to remove mass as early as possible to prevent spreading.
Cholesteatoma can be congenital or acquired. It’s the growth of keratinising squamous epithelium that causes destruction in middle ear and temporal bone. Congenital have no tympanic perforation or otorrhea with a normal tympanic membrane. Originate from epidermal cysts found medial to tympanic membrane.
Acquired are from chronic middle ear disease and retraction pockets.
Needs surgical excision. Usually from posterior.

137
Q

Mastoid surgery angles for cholesteatoma

A

Permeatal - minimal scarring, combined with otoendoscopy for better access to remove middle ear cholesteatoma otherwise difficult. Less invasive than endaural and postaural approaches. Disadvantages only useful for attic cholesteatoma with no extension to mastoid antrum. Access limited by external ear canal. Unsuitable for repair of large or anterior perforations with need to repair tympanic membrane. Graft placement harder.
Endaural - good for repairing large central perforations and posterior perforations. Easy to place grafts. Disadvantage visible scarring, difficult access to anterior margin of tympanic membrane for repair of anterior perforations.
Postaural - good access for removing cholesteatoma with posterior excision and for anterior perforations. Easy to place grafts. Disadvantage scar in post auricular area and difficulty reaching posterior perforations.

138
Q

Combined approach tympaniplasty vs modified radical mastectomy for mastoid surgery

A

CAT canal down vs Canal wall up.
CAT often need secondary operations as can’t see everything first time round need to check. Longer operation and anaesthetic time.
No difference in post op hearing
CAT maintains more normal structure and heals faster maintaining self cleaning mechanism of ear. No need for water precautions. Favoured for children and localised disease.
MRM technically easier less complication rates. Better access and visualisation of anatomy recommended for failure of CAT, extensive disease of middle ear or mastoid, labyrinthine fistula and variations in anterior sigmoid sinus anatomy. Longer healing (4months), moisture and infection problems needs regular aural toilet. Altered anatomy can cause hearing aid fitting problems etc.
Rate of reoccurance thought to be higher in CAT, disease easier to identify in MRM.

139
Q

Middle ear mechanisms to improve sound transfer

A

Ratio of tympanic membrane to stapes footplate is 17:1 so is funnel shaped
Handle of malleus is longer than incus by 1.3:1 ratio
Natural resonance of outer and middle ear for frequencies of 2000-5000Hz while tympanic membrane and ossicular chain has optimal transmission at 500-3000Hz
Buckling effect of tympanic membrane sound is enhanced via these mechanisms by ratio of 20:1

140
Q

The facial nerve

A

Has motor, sensory and parasympathetic innervation
4 functional components - two afferent : taste from anterior 2/3 of tongue via lingual nerve, chorda tympani and nervus intermedius to nucleus tractus solitarius and sensory fibres from external auditory canal and auricle
And two efferent nerves : motor to posterior belly of digastric, stylohyoid, stapedius, muscles of facial expression. And parasympathetic to lacrimal gland (greater superficial petrosal nerve) and salivary glands (chorda tympani).

141
Q

Segments of the facial nerve

A

Intracranial -22-24mm long from line to internal auditory canal
Meatal segment -8-10mm and runs from fundus of internal auditory canal to meatal foramen
Labyrinthine segment -3-5mm from meatal foramen to geniculate ganglion. Greater superficial postrosal nerve (first branch). Fallopian canal is narrowest in this section.
Tympanic -8-11mm from geniculate ganglion where the nerve makes a 40-80degrees turn and runs posteriorly across the tympanic cavity to the pyramidal Eminence. Commonest site of injury from trauma
Mastoid -10-14mm from pyramidal eminence to stylomastoid foramen
Extra-temporal - stylomastoid foramen to muscles. 5 terminal branches (temporal, zygomatic, buccal, mandibular, cervical). Pierces parotid and forms plexus of nerves called pes anserinus

142
Q

Terminal branches of facial nerve

A
Superior maxillary 
Taste fibres
Temporal 
Zygomatic 
Intraorbital
Buccal
Mandibular 
Cervical
143
Q

Facial nerve dehiscence

A

Congenital facial nerve dehiscence is found in around 15% of people
Most common site is in tympanic segment followed by second genu
Prevalence of dehiscence on histopathology of normal temporal bones is 56% although most will be encountered during surgery

144
Q

Scottish Bell’s palsy house grading

A

I - normal symmetrical function in all areas
II - slight weakness, complete eye closure with minimal effort, slight asymmetry of smile with max effort
III - obvious weakness not disfiguring. Complete eye closure but asymmetrical mouth movement
IV - obvious disfiguring weakness, unable to lift brow, incomplete eye closure and asymmetry
V - little motion, incomplete eye closure slight mouth movement
VI - no movement loss of tone, no contraction or spasm.

145
Q

Bell’s palsy

A

Commonest cause idiopathic 70%
Damage to CN VIII
Recurrent palsies occurs in 10-12% patients
Steroids only proven treatment but still controversial
Recovery depends on severity of initial palsy. Need to keep eyes closed and use eye drops to hydrate eyes

146
Q

Ramsay hunt syndrome

A
Infection with Herpes varicella zoster (herpes virus 3) 
Remains dormant in geniculate ganglion
CN VIII palsy, hearing loss and vesicles on external auditory canal, tympanic membrane and pinna
Causes pain, discharge 
Prescribe acyclovir (antiviral drug) and prednisolone (steroids)
147
Q

Waldeyers ring

A

Lymphoid tissue
Forms a ring in superior pharynx
Composed of
- adenoids (pharyngeal tonsil), Roof of posterior wall of nasopharynx
- tubal tonsils, submucosa of pharynx near pharyngeal orifice of Eustachian tube
- Palatine tonsils, sit in tonsillar fossa
- lingual tonsil, posterior third of tongue, posterior to palatoglossal arches

148
Q

Tonsil size grades

A

1- hidden from view 25%
2- with palatopharyngeal arch 50%
3- protruding into oropharynx outside tonsillar fossa 75%
4- touching tonsils
Large tonsils can cause airway obstruction resulting in sleep apnoea particularly problematic in children as lack of oxygen prevents brain development sometimes

149
Q

Quinsy

A

Abscess in mouth
Stops eating and drinking
Treated with antibiotics and must drain the abscess to prevent reoccurrence
Very painful, pussy and gross

150
Q

Indications for tonsillectomy

A

Recurrent tonsillitis:
More than 7 episodes in one year, more than 4 for 2 years, more than 2 for 3 years
Asymmetrical tonsils - can be natural anatomical difference in children but in adults can be sign of tumour
Quinsy abscess
Sleep apnoea from tonsil size
Tonsilliths
Unknown primary cause (30% patients) removal needed to discover cause of pain etc. Such as cancers

151
Q

Tonsillectomy

A

Two methods
Intracapsular - meeee technique, less painful. Dissolves tonsil. 1 in 100 grow back. Surgery takes longer to ensure full removal.
Extra capsular - traditional method using burning/dissolving tool to cut through tissue but not capsule and remove the tonsil. Easy procedure hopefully no bleeding but has higher bleeding rates.
Nearby structures- 2-2.5cm away from carotid artery must not hit causes lots of bleeding

152
Q

Neuromuscular supply to palatine tonsils

A

Nerve supply- glossopharyngeal multiple small neurones
Arteries - facial branches, tonsillar branch, ascending palatine branch.
Lingual branches, dorsal lingual artery
Ascending pharyngeal artery
Maxillary branches, lesser descending palatine artery
Veins - lingual and pharyngeal drain into external jugular vein
Lymphatic drainage to level 2

153
Q

Oropharyngectomy

A

Removes the pharynx and tonsil
Question mark incision
Removes constrictor muscles
Parapharyngeal fat used as limit to protect carotid artery
Paravertebral fascia cut 1 Cm margins
Main complications bleeding and pain
For t1 and 2 tonsil SCC with N1, debatable for N2a disease
Improved swallowing outcomes, avoids chemotherapy and radiotherapy
Surgery techniques- robot, transoral laser, direct vision with diathermy or harmonic scalpel

154
Q

Tongue based mucosectomy

A
Used for unknown primary cause of tonsil removal and lingual tonsillectomy 
Detects 20% underlying causes
Surgical techniques-
Robot
Endoscopic
Main complications pain and bleeding
155
Q

Thyroid gland

A

Largest endocrine gland in body
Deep to sternohyoid and sternothyroid
C5-t1
Right and left lobes with isthmus lying over trachea above or below pyramidal lobe
Berry’s ligament attaches thyroid to second and third rings
Thyroid capsule
Blood supply- external carotid artery branches (superior thyroid artery) and inferior thyroid artery from subclavian.
Lymphatics - deep cervical, paratracheal and pretracheal

156
Q

Recurrent laryngeal nerve

A

Vagus nerve branch
Passes under arch of aorta on left and subclavian on right
Runs in tracheoesophageal grove
Less than 1% non recurrent

157
Q

Parathyroids

A
Located outside thyroid capsule
Superior and inferior 
Supplied by inferior thyroid artery
Veins drain into thyroid venous plexus
Third superior and fourth inferior brachial arches 
Location varies
158
Q

Complications of thyroid surgery

A
Bleeding
Infection
Dysphonia from recurrent laryngeal nerve injury
Hypocalcaemia
Long term medication
159
Q

Parotid and submandibular glands and surgery

A

Tightly bound in fascia if inflamed will be very painful
Incision made in front at least 2 Cm under mandible to avoid facial nerve running through parotid
Damaging nerve causes ear numbness

160
Q

Perotid gland

A

Largest salivary gland
Irregular shape
Sits in parotid bed anteroinferior to External ear canal between ramus of mandible, styloid and mastoid process
Covered with parotid sheath (part of investing layer of fascia)
Parotid duct passes superficial to masseter before turning inward over anterior edge and empties opposite second maxillary molar
Structures within:
Facial nerve branches, Pes ancerinus, temporal, zygomatic, buccal, mandibular and cervical.
Retro mandibular vein and external carotid artery.
Neurovascular supply - great auricular nerve c2-3 supplies parotid sheath and overlying skin. Auriculotemporal nerve V3 brings parasympathetics from Otic ganglion (IX). Sympathetic supply via carotid artery. Lymph nodes lie within gland or drains into superficial and deep cervical nodes

161
Q

Complications of parotid gland surgery

A

Bleeding
Infection
Salivary fistula - duct drains into wrong place
Frey’s syndrome - damage to or near parotid glands makes them very sweaty etc.
Facial nerve injury
Numbness

162
Q

Submandibular glands

A

Located in submandibular triangle
Lies along body of mandible and wraps around myelohyoid muscle
Duct arises from deep portion and runs 5cm to open through 3 orifices next to frenulum on floor of mouth
Lingual nerve loops under duct
Arterial supply from submental artery
Parasympathetic from VII via chorda tympani and lingual nerve which synapse in submandibular ganglion
Sympathetic supply is from superior cervical ganglion
Complications of surgery
Bleeding, infection, nerve damage to lingual, hypoglossal and marginal mandibular nerve.

163
Q

Function of the larynx

A

Swallowing
Speech
Straining - close off glottis
Protects tracheobronchial tree and lungs

164
Q

The larynx

A

Composed of epiglottic cartilage, thyroid cartilage, cricoid cartilage, arytenoid cartilages (pair), cuneiform cartilages (pair), corniculate cartilage (pairs)
4.5cm length in men
3.5cm in women
Ligaments- extrinsic, hyp-epiglottic connecting epiglottis to back of hyoid, thyrohyoid membrane, crocothyroid ligament between thyroid cartilage and cricoid cartilage. Cricotracheal ligament links cricoid to first tracheal ring.
Intrinsic - cricothyroid ligament from cricoid cartilage to thyroid cartilage. Quadrangular membrane - between anterolateral arytenoid cartilage and lateral epiglottis.
Muscles - extrinsic, attach larynx to neighbours. Sternothyroid depresses larynx connects sternum to thyroid cartilage. Thyrohyoid elevates larynx connects hyoid to thyroid. Inferior pharyngeal constrictor.
Intrinsic - move laryngeal cartilages against each other. Posterior cricoarytenoid muscle abducts cords. Lateral cricoarytenoid muscle adducts cords. Inter arytenoid muscle adducts cords. Thyroarytenoid muscle relaxes cords. Cricothyroid muscle tenses cords. Vocalis muscle for fine adjustment.
All besides the cricothyroid are supplied by the recurrent laryngeal nerve.
Cricothyroid supplied by the external laryngeal nerve - branch of superior laryngeal nerve.

165
Q

Supraglottis, glottis and subglottis definitions

A

Supraglottis - from free edge of the epiglottis and aryepiglotic folds to true vocal folds
Glottis - space between true vocal folds
Subglottis - inferior border of vocal folds to inferior boarder of cricothyroid.

166
Q

Phonation

A

Results from fundamental tone produced at true vocal folds
Initial speech signal is then modified by resonating chambers of upper aerodigestive tract
Two theories -
Myoelastic-aerodynamic theory - during expiration airflow is unidirectional and vocal cords vibrate causing intermittent interruption of airflow directing air to an alternating airflow modulators
Neuromuscular theory - now disproved, suggested a vibratory cycle is initiated by central neuronal impulses via the vagus to the larynx. Rate of impulses theoretically determined the frequency of VC vibration.

167
Q

Myoelastic-aerodynamic theory of speech

A

during expiration airflow is unidirectional and vocal cords vibrate causing intermittent interruption of airflow directing air to an alternating airflow modulators
Step 1- vc adducts and undergo appropriate tension
2- pressure in subglottis increases due to muscular and passive forces of exhalation
3- glottis is forced open
4- following outflow of air the pressure decreases and VC start to return to position via myoelastic forces
5- forces are enhanced by Bernoilli Effect (negative pressure in channel secondary to airflow) and therefore VCs are sucked back.
6- creates sawtooth waveform, jagged lines not smooth.

168
Q

Deglutination

A

Function of swallowing:
Food bolus passage into oesophagus
Disposes of mucus loaded with dust and bacteria from respiratory passages
Eustachian tube is opened equalising pressure either side of ear drum (doesn’t contribute to swallowing but happens as consequence)
Has oral, pharyngeal and oesophageal phases, oral is voluntary but completed with involuntary reflex from pharyngeal stimulation. Coordinated in deglutination centre in medulla near vagal nucleus and respiratory centres .

169
Q

Oral phase of mastication

A

V and VII moistened by parasympathetics to salivary gland
Hypoglossal XII for movement - food bolus pushed through oropharyngeal isthmus by pressure of tongue against the palate, assisted by muscles of mouth floor XII.

170
Q

Involuntary reflex pharyngeal phase mastication

A

Receptors are proprioceptive with afferent limb via IX and efferent IX And X
To prevent regurgitation oral, nasal and laryngeal openings must be closed
- nasopharynx closed by elevation of soft palate IX and X, tensor palati V opens Eustachian tube
- oropharyngeal isthmus blocked by contraction of palatoglossus IX and X and Instrinsic muscles of tongue XII.
Laryngeal protection complex, closure of sphincter mechanism by aryepiglottic folds, walls of laryngeal vestibule by thyroepiglottic muscles and vocal cords apposed by cricoarytenoid and inter arytenoid muscles
Larynx gets tucked behind overhanging mass of tongue
Epiglottis guides the food bolus away from laryngeal opening.

171
Q

Role of the epiglottis

A

As food bolus reaches epiglottis is tipped backwards against the pharyngeal wall and momentarily halted here
Larynx elevated and pulled forward drawing with it epiglottis now stood erect
Guides food into two streams along both piriform fossi and away from laryngeal orifice in middle
Finally epiglottis flaps back as cover over laryngeal inlet but this occurs only as main bolus passed beyond it.

172
Q

Role of vocal cords

A

Adduct during deglutination
Patients who have small or missing epiglottic cartilage due to disease often have a safe swallow while those with damaged vocal cords often aspirate suggesting a vital role for the vocal cords in protecting the airway

173
Q

Oesophageal phase swallowing

A

Inferior pharyngeal constrictor constricts and cricopharyngeus relaxes allowing bolus to cross the pharyngo-oesophageal junction.
Where further involuntary striated and smooth muscle propels food bolus (X)

174
Q

Protective function of larynx

A

Superior laryngeal nerves don’t cross innervate
Internal laryngeal nerve supplies supraglottic mucosa
Stimulation produces strong adductor response and laryngeal spasm with inhibition of abductors. Further stimulation of all major cranial afferents, special sensory and spinal somatic sensory nerves
Can cause reflex apnoea and laryngospasm

175
Q

HPV associated papilloma treatment

A

Needs multiple surgery’s often reoccurring
Most isolated in larynx but if spread to lungs can be fatal
Burn or dissolve nodules

176
Q

Laryngeal papillomas

A

Affects all ages characterised by recurrent wart like benign epithelial growths I’m airways
HPV can be associated
Healthy people can fight HPV virus before symptoms but some can’t and it persists and remains dormant and papillomas form on reactivation.
If lung parenchyma becomes involved can be fatal life expectancy less than 10 years
HPV vaccine protects against subtypes 6,11,16 and 18 and is given to females aged 12-20 to prevent cervical HPV infection before girls become sexually active.

177
Q

Vocal cord nodules

A

Vocal cord abuse, poor phonation techniques or rarely coughing
Children, singers, teachers
Raspy voice
Junction of anterior 1/3 and posterior 2/3 most common for nodules
Treatment is speech and language therapy and voice rest initially.
Should try to avoid surgery as causes lots of scarring

178
Q

Reinkes oedema

A

Benign
Smoking and reflux main causes
No treatment but can be aspirated
Speech and language therapy used and removing the precipitating factor can help but usually comes back

179
Q

Vocal cord cancer

A

Needs to be surgically removed
Usually doesn’t spread (5% do)
Has husky voice needs radio and chemotherapy is too strong for larynx so will likely need to remove whole thing not just vocal cord tumour

180
Q

The acute airway

A

Stridor - proximal airway partial obstruction results in turbulent airflow causing harsh vibratory sound
- inspiratory - expiratory - biphasic
Stertor - sonorous respiration due to partial obstruction of upper airway
Worrying features - patient agitation, hypoxia and increased breathing work
Management - supplemental oxygen, intravenous access for antibiotics and steroids to widen airways, sit the patient up, Heliox if appropriate (21% oxygen 79% helium), surgery tracheostomy, intubation oral or nasal.

181
Q

Laryngectomy of cancers

A

Most laryngeal cancers are treated with radical radiotherapy or chemotherapy
Surgery reserved for more severe cases or recurrence.
Post surgery problems -
Humidification, use Buchanan Bib and Swedish nose fitted to humidify air.
Phonation problems put valve in place to act as cords, electrolarynx to replace voice, oesophageal speech (burp speech)
Deglutination problems reconstruction needed, substituted by NG tubes, PEG/RIG, strictures may occur.
Fistulae may arise
Strictures may also arise

182
Q

Oesophageal speech

A

Best result from laryngectomy
Air swallowed into cervical oesophagus
Air is immediately expelled out causing vibrations of pharyngeal mucosa
These vibrations along with tongue in oral cavity cause articulations
Very difficult to learn and only 20% patients succeed
Speak in short bursts
Two methods to pumped into cervical oesophagus - injection and inhalation

183
Q

Fistulae

A

Pharyngocutaneous fistulas PCF are the commonest complication following laryngectomy
Incidence varies from 3-65%
Radiotherapy most important risk factor
Most close spontaneously
Can use salivary bypass to avoid irritating them while they heal
High morbidity and can cause fatal complications
Delaying oral feeding, impact on quality of life and prolonged hospital stays often
Treatment-
Non surgical - delay oral feeding, swabs, freshening of fistula edges, broad spectrum antibiotics performed with local or free flaps
Surgical - local or free flaps and fistula tubes

184
Q

Define stricture

A

Abnormal narrowing of a body passage especially tube or canal
May be due for example to scar tissue or tumour

185
Q

Cerebrovascular considersations

A

Extremely high demand for oxygen and nutrients - human brain is 2% of bod weight, receives 15% Co, 20% total body oxygen consumption and 25% total body glucose utilisation
Disease and stroke are among major causes of death
Damage to vascular supply to spinal cord leads to either motor paralysis or sensory loss and is major cause of disability

186
Q

Cerebrovascular accidents

A

Results from ischaemia to part of brain or haemorrhage into brain results in death of brain cells
Approx 100000 UK annually cases
Third most common death cause
First cause of disability
25% initial stroke die within one year
50-75% will be functionally independent
25% will be permanently disabled
Physical, cognitive, emotional and financial impact
Risk factors - hypertension, smoking, high cholesterol, heart disease, oral contraceptive, sickle cell disease, hypercoagulability, diabetes, TIA, atrial fibrillation, physical inactivity, asymptomatic carotid stenosis, age, gender (females more common), family history, medical history

187
Q

Circle of Willis

A

Encloses optic chiasm, pituitary stalk and mammillary bodies
Oculomotor nerve exits between posterior cerebral and superior cerebellar arteries
Vertebral arteries of two sides unite to form basilar artery at ponto-medullary junction
Root of abducens nerve and initial segment of anterior inferior cerebellar artery can also be found here
Complete symmetric circle found in less than 50% individuals
In some cases 60% at least one component is relatively hypo-plastic and diminished in capacity to provide collateral flow - multiple hypo-plastic anomalies.

188
Q

Venous plexuses

A

Pterygoid plexus

Basilar venous plexus

189
Q

Cerebral veins

A
Superior saggital sinus vein
Superior cerebral 
Superficial middle cerebral
Sigmoid sinus
Internal jugular 
Transverse sinus 
Mostly drain through internal jugular vein. Blood normally encourages to right IJV due to natural anatomical asymmetry of transverse sinuses
190
Q

Branches of aorta

A

Ascending aorta - right and left coronary arteries
Arch of aorta - brachiocephalic artery (splits into right common carotid and subclavian), left common carotid and left subclavian.
Descending/thoracic aorta - level at t4-12 gives off several pairs - bronchial arteries, mediastinal, oesophageal, pericardial, superior phrenic. Intercostal and subcostal originate throughout length of thoracic aorta.
Abdominal aorta - slightly to left of midline. Enters abdomen at t12. inferior phrenic arteries at t12 supply diaphragm. Coeliac artery also t12, superior mesenteric l1, middle suprarenal arteries l1 to adrenal glands, renal arteries L1/2, gonadal arteries L2, inferior mesenteric L3, median sacral artery L4 supplies coccyx, lumbar vertebrae and sacrum. Lumbar arteries two pairs (4) between L1-4 supply abdominal wall and spinal cord.

191
Q

Coeliac trunk branches

A

Emerges from aorta at t12 and divides into three approx 1cm after into left gastric, splenic and common hepatic arteries. left gastric is smallest and gives rise to oesophageal branches and continues along lesser curve of stomach to anastomoses with right gastric artery. Splenic runs along superior margin of pancreas containing within splenorenal ligament. Gives off left gastroepiploic (greater curve of stomach), short gastrics (5-7 small branches supplying stomach fundus) and pancreatic branches (body and tail pancreas). Terminates into 5 branches supplying spleen.
Common hepatic is sole arterial supply to liver only branch to pass right from coeliac. Divides into proper hepatic and gastroduodenal.
Proper hepatic gives rise to right gastric (pylorus And lesser curve of stomach), right and left hepatic (lobes of liver), cystic branch from right hepatic (gall bladder)
Gastroduodenal branches into right gastroepiploic (greater curve stomach) and superior pancreaticoduodenal (head of pancreas)

192
Q

IMA branches

A

Arises from aorta at L3. Splits into left colic, sigmoid and superior rectal artery. Left colic supplies distal 1/3 transverse colon and descending colon. Travels anteriorly to psoas muscle before dividing into ascending and descending branches.
Sigmoid arteries supply descending and sigmoid colon, typically 2-4 branches the highest called superior sigmoid artery
Superior rectal artery supplies rectum divides at S3 into two branches supplying either side. Anastomoses with middle and inferior rectal arteries.

193
Q

SMA branches

A

Arises from abdominal aorta at L1 is posterior to stomach pylorus, splenic vein and neck of pancreas and anterior to left renal vein, uncinate process of pancreas and inferior duodenum.
First branch is inferior pancreaticoduodenal artery supplies head, uncinate process of pancreas and duodenum
Jejunal and Ilieal arteries form arcades supplying small intestine
Middle and right colic arteries supply ascending and transverse colon
Ileocolic artery final major branch gives rise to branches in ascending colon, appendix, Cecum and ileum.

194
Q

Stomach blood supply and surgery

A

Mostly from coeliac trunk has multiple anastomoses excellent for surgery as doesn’t matter if few arteries are tied off or damaged due to such rich blood supply

195
Q

SMA landmark in surgery

A

Pancreas in curvature of duodenum, SMA runs underneath pancreas and over duodenum

196
Q

Which artery is tied off for appendix removal

A

Find cecum, follow tenea coli and tie off iliocolic branch supplying appendix.

197
Q

May Thurner syndrome

A

On the right side the common iliac artery crosses over the common iliac vein compressing it - mostly in women due to less space - causing reduced venous drainage

198
Q

Aetiology and examination of aortic aneurysms

A

Congenital - connective tissue disorders like Ehlers Danlos
Acquired - most common cause. Atherosclerosis, inflammatory diseases (vasculitides), infective (mycotic like syphilis, bacterial), trauma.
Signs - internal bleeding on centre and sides where blood fills retroperitoneal space
Blood pressure often higher than normal on onset however as blood loss increases blood pressure decreases
9/10 patients die before making it to hospital.
If loose peripheral pulse is very very bad.
Abdominal palpation very gentle so as not to worsen bleed, not very effective for finding size of aneurism or location due to pain.
Jugular bruit listened for- large aneurism can rupture of IVC as well as causing fistula. Not very common. Creates internal circulation so very high central blood pressure but very low peripheral.
Management - oxygen administered, often die of hypoxia before blood loss especially if respiratory problems. IV access, FBC, U&E, LFT, amylase, clotting profile, crossmatch blood type or use o neg always available in hospitals.
2222 rupture call
Hypotensive haemostasis to keep patient talking to ensure brain perfusion. Restrictive approach of IV fluid administration to maintain MABP high enough for organ perfusion but lower than normal to limit bleeding. Any sudden blood flow changes can cause clots and death. Use large diameter short length cannula for better and faster flow. Cannot resuscitate anyone with a central line!!!!?
Imagery of aneurism shows if calcium and thrombus present in vessel or if hollow, if hollow is weaker and more likely to rupture due to more wall stress.

199
Q

Open abdominal aortic aneurism repair techniques

A

Most are infer renal and end before aortic bifurcation - helpful as kidneys can’t live without blood for more than 20 mins blood but can occlude aorta in between for hours. Clamp section while fixing to stop bleeding. Stitch a tube graft to replace aneurism section.
Endovascular Aneurism repair EVAR repair technique via the groin using stents with grips to fix aorta. Les invasive. Must ensure stents seal fully and don’t block kidney vessels.

200
Q

Thoracoabdominal branched aortic aneurism operations

A

Starts in chest ends in/near pelvis. Very large graft needed and must be pre made cannot repair in emergency ruptures due to blocking all vessels stemming from aorta.
Needs a pre made specific graft with holes matching exact places of vessel branches
Once in place then extra stents added to branching vessels to reinforce stents and hold in position
Can do in open repair - very large amount of tubing to replace entire aorta and branches.

201
Q

Limb ischaemia

A

Abrupt cessation of arterial blood flow to extremity resulting in hypoperfusion of tissue threatening limb viability
Two types - acute and chronic
Risk factors - high blood cholesterol, smoking, diabetes, family history
Aetiology - embolic - arrhythmia, cardiomyopathy, iatrogenic causes. Or thrombotic - atherosclerosis or bypass graft. Or traumatic.
Causes pain. If pain at rest/can’t lie Down then occlusion is bad.
Clinical examination, ECG, Bloods, imagining needed.
Signs - pain, pallor(pale), perishing with cold, pulselessness, paraesthesia or reduced sensation/numbness, paralysis or reduced power.
Imaging angiogram shows shrivelled branches from collateral vessels especially in smokers - 1/3 patients improve collateral vessels just from stopping smoking.
Surgery is last resort only if becomes severe
Open surgery - very common, clamp vessel make hole in artery open, find clot and remove.
Can use bypasses for chronic occlusion
Aorto-bifemoral bypass, femoral-popliteal bypass, femoral-femoral crossover. Named after donor and recipient vessels.

202
Q

Lower limb major vessels

A

Aorta bifurcates at L4 into common iliac vessels. Branches at pelvic brim into internal and external iliac vessels. External iliac supplies leg, passes through femoral triangle as common femoral artery, then to deep and superficial femoral supplying thigh. Travels through adductor hiatus canal becomes popliteal artery at posterior calf. Anterior tibial branches off and posterior tibial continues gives off branch of peroneal artery. Anterior tibial becomes pedal arch arteries and posterior tib becomes dorsalis arteries which anastomoses together in foot.

203
Q

Endovascular surgery and thrombolysis

A

Catheter directed thrombolysis - pass catheter over clot, leave in overnight, dissolves blood clot and saves having major surgery
Use stent to stretch vessel rather than bypassing
Embolyectomy or lysis of clot to dissolve it used for acute blockages. Lysis used early on or embolyectomy in late stages
Mechanical thrombectomy - break up clot into small pieces and removed.
Ultrasound enhanced thrombolysis

204
Q

Reperfusion syndrome

A

Occurs as result of blood flow going back into previously damaged disuse causing rhabdomyolysis and compartment syndrome
Causes extreme pain and needs fasciotomy - cut into compartment and relieve pressure for few days until muscle swelling decreases and then can be sewn up.

205
Q

Arterio-venous fistulas

A

Creating a fistula between artery and vein gives vein constant blood flow making it more stable for IV for dialysis etc. Allowing vein to last longer before damage and needing to find new vein area to inject

206
Q

Panda eyes cause

A

Blood vessel injury in scalp - blood pools in eye lids

207
Q

Blood supply of scalp

A

Mostly superficial branches from external carotid artery - supratrochleal artery, supraorbital, superficial temporal artery frontal, parietal branches, posterior auricular artery, occipital artery.

208
Q

Arteries of dura mater and skull

A

Middle meningeal artery most important
Artery to trigeminal nerve ganglion
Internal maxillary artery
Often bones at side of skull are thinner and so can cause potential damage to middle meningeal vessels is possible

209
Q

Extradural haematoma

A

Extradural haematoma caused from middle meningeal layer bleeding needs surgical removal - craniotomy, skull removed and blood clot syphoned off, very successful healing rate

210
Q

Epidural vs subdural haemotoma

A

Epidural - outside dural lining, usually caused by skull fracture. Rapidly expanding with arterial blood. Dura pushes away by haemotoma
Subdural - caused by rupture more likely venous. Beneath dura. Slowly expanding with venous blood. Dura still attached to skull, can’t cross falx.

211
Q

Route of blood to brain

A

Anterior - carotid arteries gives rise to middle and anterior cerebral arteries, supplies frontal, parietal, temporal lobes, nasal ganglion, part of dienecephalon (thalamus and hypothalamus)
Posterior - off subclavian artery in through vertebral foramen. Gives rise to vertebral arteries - basilar artery supplies mid and lower temporal and occipital lobes, cerebellum, brainstem And part of diencephalon.
Circle of Willis - anastomoses of anterior and posterior cerebral circulation

212
Q

Internal carotid artery segments

A

From base to terminals
Cervical, petrous, lacerum, cavernous, clinoid, ophthalmic, communicating
Travels through carotid canal

213
Q

Anterior cerebral artery involvement for stroke effects

A

Brain stem occlusion
Contralateral - weakness of proximal upper extremity - sensory and motor deficits of lower extremities
Urinary incontinence
Sensory loss (discrimination, proprioception)
Contralateral grasp and suckling reflexes may be present
Apraxia - loss of ability to carry out familiar purposeful movements in absence of sensory or motor impairment
Personality changes - loss of spontaneity, loss of interest in surroundings
Cognitive impairment

214
Q

Middle cerebral artery involvement of stroke effects

A

Contralateral weakness - hemiparesis; hemiplegia
Contralateral hemianaesthesia
Loss of proprioception, fine touch and localisation
Dominant hemisphere: aphasia (loss of speech) from damage to Brochas area
Non dominant hemisphere: neglect of opposite side, anosognosia - unaware or denile of neuro deficit
Homonymous hemianopsia - defective vision or blindness right or left halves of visual fields of both eyes

215
Q

Posterior cerebral artery involvement in stroke effects

A
Range from alert to comatose
Unilateral or bilateral sensory loss
Contralateral or bilateral weakness
Dysarthria - impaired speech articulation
Dysphasia - difficulty in swallowing
Hoarseness
Ataxia, vertigo
Unilateral hearing loss
Visual disturbances (blindness, homonymous hemianopsia, nystagmus, diplopia
216
Q

Cerebrovascular accident classification

A

Ischaemic - thrombotic, embolic
Haemorrhaging - intracerebral or subarachnoid - aneurism berry or saccular
Diagnosed by neuro assessment, ultrasound, CT to identify size, location, differentiate type, CT angiography for vasculature, MRI for greater specificity, angiography is best.

217
Q

Ischaemic stroke

A

Inadequate blood flow 85% all strokes
Extend of damage depends on rapidity if onset, size of lesion, presence of collateral circulation
Symptoms progress fast in first 72 hours as infarction and cerebral oedema increase
Types - thrombotic or embolic
Major cause atherosclerosis
Common atherosclerotic locations at vessel junctions such as aortic bifurcation etc.
Stents can stabilise wall or remove the clot.
Antiplatelets, modify risk factors, carotid endarterectomy (strip away and remove plaque use shunt to maintain blood supply) if occlusion more than 70% stenosis, complications can include stroke, bleeding, infection, cranial nerves X pharyngeal and superior laryngeal, XII hypoglossal nerve, XI accessory nerve, glossopharyngeal IX at risk

218
Q

Transient ischaemic attach TIA

A

temporary loss of neurological function
Caused by ischaemia of one of vascular territories of Brian - microemboli with temporary blockage of blood flow
Lasts less than 24 hours often less than 15 mins
Most resolve within 3 hours
Warning sign of progressive Cerebrovascular disease

219
Q

Hemorrhagic stroke

A

15% all strokes
Result from bleeding into Brain tissue
Intracerebral - rupture of vessel, hypertension most important cause, vascular malformations, coagulation disorders, anticoagulation, trauma, brain tumour, ruptured aneurisms. Sudden onset symptoms with progression. Neurological deficits, headache, nausea, vomiting, decreased LOC and hypertension, prognosis is poor 50% die within weeks 20% functionally independent at 6 months. Not most common type.
Subarachnoid - intracranial bleeding into space between arachnoid and pia mater membranes. Commonly caused by rupture of cerebral aneurism saccular or berry 20/30mm size majority occur in Circle of Willis. Other causes arteriovenous malformation, trauma, drug use. Higher incidence in women and elderly. Neurological deficits, headache, nausea, vomiting, decreased LOC and hypertension. Poor prognosis 50% die within weeks 20% functionally independent at 6 months. Warning symptoms sudden headache or loss of consciousness, sickness, stiff neck, seizures. Can clip berry aneurisms, wrap saccular to strengthen and prevent bursting. Insert coil into berry etc. Or treat underlying AV malformation

220
Q

Arterial venous malformations

A

Connections between arteries and veins in brain

221
Q

Kidney function

A

Urinary excretion
Fluid, electrolyte and acid base balance
Vitamin D metabolism
Denim and erythropoietin production

222
Q

Development of kidneys

A

Closely related to reproductive system particularly in early stages but develops slightly ahead of reproductive system. Consists of kidneys, ureters, bladder and urethra. The intermediate mesoderm called the urogenital ridge gives rise to all structures.
Develops from three overlapping sequential systems- pronephros, mesonephros and metanephros.
Pronephros - development begins at cervical region of embryo in fourth week of development. Segmented divisions of intermediate mesoderm form tubules called nephrotomes. 6-10 pairs of nephrotomes are formed these joint to form the pronephric duct which extends from the cervical region to the cloaca of the embryo. Non functional and regresses completely by the end of week 4.
Mesonephros - develops caudally (inferiorly) to pronephros. Presence of pronephric duct induced nearby intermediate mesoderm in thoracolumbar region to form mesonephric tubules which receive capillaries from dorsal aorta allowing filtration of blood which drains into mesonephric duct (continuation of pronephric duct). Act as primitive excretory system in embryo. Most tubules regress by 2 months. Also sprouts the ureteric bud caudally which induces the definitive kidney.
Metanephros - forms definitive kidney. appears in week 5 and is functional in 12th week. Ureteric bud forms mesonephric duct and contacts caudal region of intermediate mesoderm (metanephric blastema) to form metanephric system. Has two parts - collecting system derived from ureteric bud creating the ureter, renal pelvis, major and minor calyces and collecting tubules terminating at DCT. If splits too early may result in two ureters or renal pelvises. And the excretory system derived from metanephric blastema, forming the nephrons (bowmans capsule, glomerulus, PCT, loop of henle and DCT).
Initially develops in the pelvic region before ascending to abdomen. In pelvis receives blood supply from pelvic branch of aorta and as it ascends new arteries supply. Pelvic vessels regress but can persist as accessory arteries.

223
Q

Development of urethra and bladder

A

Cloaca forms bladder and urethra. In weeks 4-7 cloaca is divided into two by uro-rectal septum. Urogenital sinus (anterior) forms bladder and urethra as well as some reproductive tract in females and prostatic, membranous and spongy urethra in males. Anal canal posteriorly initially drained by allantois but this is removed in foetal development and becomes fibrous cord (urachus). Remnant of this found in adults - median umbilical ligament connecting bladder to umbilicus.
As bladder develops it absorbs caudal parts of mesonephric ducts (wolffian ducts) becoming trigone of bladder. Ureters enter at base of trigone.

224
Q

Kidney migration

A

Kidneys migrate upwards in foetal growth within abdomen from weeks 6-10
Migration halts on reaching suprarenal glands
Mesonephric artery at that point remains the renal artery

225
Q

The ureter

A

Bilateral muscular conduits which communicates urine from kidney to bladder
25cm length
Retroperitoneal with abdomen and pelvic parts
Closely related to many other structures
Structure - transitional epithelium, lamina propria, longitudinal And circular smooth muscle, adventitia encompasses blood vessels and lymphatics which travel with the ureter.
Blood supply very easy to damage so surgeons try to avoid touching or handling ureter unless absolutely necessary

226
Q

Course of the ureter

A

Abdominal - Starts at pelvic ureteric junction PUJ posterior to renal artery and vein L2 left lower on right. Runs inferiorly along medial aspect of psoas major
Crosses under gonadal vein at level of inferior boarder of kidney
Run medial towards sacroiliac joint
Enter true pelvis by crossing anterior to bifurcation of common iliac artery 5cm apart
Pelvic - runs posteriorly and inferiorly on lateral walls of pelvis anterior to iliac vessels curving anteriomedially, superior to levator ani to enter bladder. Runs obliquely at left and right ureteric orifice. In the male it lies above seminal vessels and crossed superficially by vas deferens
In females passes above lateral fornix of vagina lateral to cervix and below broad ligament and uterine vessels

227
Q

Relations to ureter

A

Abdominal
Posteriorly - psoas major, genitofemoral nerve, common iliac vessels, tops of L2-5 transverse processes
Anterior - right ureter - duodenum, gonadal vessels, right colic vessels, iliocolic vessels
Left ureter - gonadal artery, left colic vessels, loops of jejunum, sigmoid colon and mesentery.
Laterally IVC for right ureter
Pelvic
Posterior - sacroiliac joint and internal iliac artery
Inferior - males seminal vessels and females lateral fornix of vagina
Anterior - male ductus deferens and female uterine artery in broad ligament
Lateral - female - cervix

228
Q

Ureter blood supply

A

Segmental blood supply

Arterial branches

229
Q

Horseshoe kidney

A

Renal fusion is a congenital disorder affects 1 in 400 people, more commonly men.
Kidneys isthmus fuse to form horseshoe shape during development and are blocked by the inferior mesenteric artery at L3 from rising higher.
At increased risk for kidney stones, renal cell carcinoma and hydronephrosis.

230
Q

Ureter blood supply and innervation

A

Arterial blood supply varies along course
Upper part closest to kidneys - renal arteries
Middle part from common iliac arteries direct branches from abdominal aorta and gonadal arteries (testicular/ovarian arteries)
Lower part closest to bladder - internal iliac arteries as well as superior vesicle artery, uterine artery in women, middle rectal artery, vaginal artery in women and inferior vesicle artery in men only.
Nerve supply ureteric plexus Derived from adjacent autonomic plexuses (renal, aortic, superior and inferior hypogastric). Primary sensation from T12-L2.

231
Q

Narrowing of ureter

A

Pelvic ureteric junction
Pelvic brim
Vesicoureteric junction

232
Q

Urinary bladder

A

Temporary store of urine
Hollow muscular viscus with distensible walls
Dynamic anatomically depending on content, state of adjacent organs and patient age - when full can extend to level of umbilicus.
Structure
Transitional epithelium, lamina propria, submucosa, detrusor muscle - criss cross arrangement of bundles. In chronic obstruction lead to trabeculated appearance of bladder
Adventitia. At bladder neck muscle fibres form involuntary internal sphincter
In males fibres are continuous with fibromuscular tissue of prostrate gland
Ureteric orifices emerge at angles of the trigone
Apex points toward pubic symphysis, superior surface covered by peritoneur, body is between apex and fundus. Base is posterior wall. Trigone area between ureteric orifices and internal urethral orifice (least mobile part). UOs connected by intraueterine ridge. Males base in rectum. Females related to anterior wall of vagina and upper cervix. Bladder neck is where inferolateral wall and base meet and is pierced by urethra. In females is above urethra in connective tissue of vaginal wall.

233
Q

Ureteroscopy and laser of kidney stones

A

X rays used for imagery

Ureteroscope inserted through urethra, bladder and up through ureter to kidneys/site of stone

234
Q

Bladder blood supply

A

Branches of internal iliac arteries
Superior vesicle arteries supply anterior inferior parts
Males inferior vesicle arteries supply fundus and neck
Females vaginal arteries replace inferior vesicle arteries
Obturator and inferior gluteal arteries also supply
Veins do not follow arteries - males vesicle venous plexus combined with prostatic venous plexus and drains across pelvic floor to internal iliac veins. Females has similar plexus communicating with veins at base of Broad ligament. Drains through inferior vesicle veins into internal iliac veins but may drain through sacral veins internal vertebral venous plexus of Baston.
Lymphatics - superior surface of bladder drains to external iliac lymph nodes
Fundus to internal iliac nodes
Neck may drain to sacral or common iliac nodes

235
Q

Suprapubic catheter

A

Used in emergency urinary retentions when a urethral catheter is not possible
Inserted into bladder using incision through stomach few inches below naval. Done under local anaesthetic or light general.
Must first confirm urinary retention using bladder scan, ultrasound
Identify two finger breadths above symphysis pubis in midline and infiltrate below skin to include fascia. Use needle to confirm urine can be aspirated from bladder - needle needs to be perpendicular to skin.

236
Q

Prostrate gland

A

Secretes alkaline fluid 30% of ejaculate
Contains some smooth muscle which aids ejection
Walnut sized surrounding prostrate
Superior aspect closely related to neck of bladder
Anterior surface is muscular and continuous with urinary sphincter
Posterior surface related to rectum
Inferior lateral surfaces related to levator ani
Lobes in relation to ureter- anterior, fibromuscular continuous with urethral sphincter with little glandular tissue. Isthmus. Posterior lobe, inferior to ejaculatory ducts. Lateral lobes are largest. Middle related to neck of bladder and only significant in benign prostatic enlargement. Can be further divisions for grading of prostrate etc.

237
Q

Prostrate gland vasculature and innervation

A

Branches of internal iliac arteries, inferior vesicle, internal pudendal and middle rectal
Venous - dorsal venous plexus drain into internal iliac veins, communicates with vesicle VS and vertebral venous plexus.
Lymphatics to internal iliac
Innervation - pelvic sphlanic nerves S2-4 parasympathetics
Inferior hypogastric plexus sympathetic

238
Q

Urethra male

A

Parts -

Prostatic, membranous, bulbous, penile

239
Q

Polycystic kidney disease

A

Inherited disorder causes fluid filled cysts form in kidneys
May impair function and eventually cause failure
Fourth leading cause of failure
Increases likelihood of developing cysts in liver and other complications

240
Q

Describe kidneys

A

Bean shaped organs retroperitoneal at level t12-l3
Left higher than right due to liver on right hand side
Renal capsule surrounded by peri-renal fat, then renal fascia and pararenal fat.
Supplied by renal arteries from aorta at L1 just underneath superior mesenteric artery branching. Renal arteries become interlobar either side of pyramids, then interlobular then entering the cortex become afferent arterioles forming capillary nertwork ending in efferent arterioles forming peritubular network around nephrons then vasorecta supplying inner 1/3 of cortex and the medulla.
Renal vein on the left is longer due to crossing the aorta and artery on the right is longer due to crossing the IVC.
Lymph drains into paraortic lymph nodes.
Drained by renal veins (lies anterior to artery in hilum).
Cortex contains vessels and nephrons - take in and filter blood for waste metabolites. Each kidney has roughly 1 million nephrons. Blood enters renal corpuscle (aka malpighigan body) containing the glomerulus and bowmans capsule. Then passes through into renal tubules - PCT, loop of henle and DCT absorbing water, sodium and glucose (PCT), K, Cl and Na (LoH), Na, K and acid (DCT) back into blood. Passes into collecting tubules in the medulla (LoH also dips into medulla) drains into renal pyramids containing strings of nephrons tubules finally moves into renal pelvis by minor and major calyces. Small cup shaped spaces that collect fluid before it moves to the bladder. Also where extra fluid becomes urine. Exits via the ureter through the hilum of the kidney into the bladder.

241
Q

Kidney topographical anatomy

A

Anterior (left) - spleen, stomach, splenic flexure, pancreas, jejunum, adrenal glands
Posterior (left and right)- psoas, quadratus lumborum, 11/12 ribs
Anterior (right)- liver, hepatic flexure, duodenum, adrenal glands

242
Q

Types of renal surgery

A

Endourology - no incision needed, scopes passed via urinalysis tract to the kidney can take biopsies or lasers.
Open - large incision usually needed, minimal access from small incisions.
Laparoscopic/robotic - much better dexterity and visuals than human
Percutaneous nephrolithotomy PCNL - minimally invasive procedure to remove Kidney stones by small puncture 1cm through skin. Most suitable to remove stones more than 2cm in size present near the pelvic region. Need multiple x rays for imagery.

243
Q

Nephectomy

A

Indications - renal cell carcinoma, failing kidney, atrophy, living donor transplantation
Types - simple, remove kidney only
Nephrouretectomy - kidney and ureter
Radical - kidney, adrenal gland, surrounding tissue and lymph nodes
Parietal - excision of cancer within kidney for renal preservation

244
Q

Kidney stones

A

Form when urine has more crystal forming substances like Ca, uric acid and oxalate than fluid can dilate. May also lack substances preventing crystals sticking together creating stones forming and often blocking ureter. Often form at calyces but can be anywhere in kidney.

245
Q

Which side of heart is more trabeculated

A

Right side to prevent blood stasis from lower diastolic and systolic pressures in the chambers

246
Q

Conditions involving concentric and eccentric hypertrophy

A

Concentric - aortic stenosis, wall thickens And causes chamber size to decrease also
Eccentric - wall is thinner and chamber is dilated and larger overall from conditions like mitral regurgitation - blood falling back into ventricle causes stretching. Also causes atria to stretch and dilate secondary to MR. Can reduce efficiency of heart by 30% from passive emptying of the atria. Once over 4cm isn’t much to be done about remodelling the heart.

247
Q

Atrial appendages

A

Small sacs in the muscle wall of the atria
The left appendage is thinner, finger like and can be involved in clotting if the heart isn’t pumping properly
The right is broad and triangular

248
Q

Atrial dilatation secondary to mitral regurgitation

A

Atrial dilatation causes passive emptying on the atria resulting in up to 30% loss in efficiency
If distended over 4cm not much can be done to remodel the heart

249
Q

Atrial fibrillation surgery

A

Cox maze procedure
Structurally breaking up atria by suture lines can correct the arrhythmia in some cases but this can now be done without surgery can use radio frequency, microwave, laser, high-intensity focused ultrasound and cryothermia incorporated into devices in order to create some of the lesions of Cox maze procedure without actually cutting atrial walls.

250
Q

Heart failure and resynchronisation surgery

A

Pacemaker implanted - used to synchronise chambers again

251
Q

Where are septal leaflets found

A

Folds directly from the septum in the right ventricle onto the tricuspid valve only in this ventricle not found in the left so is good anatomical indicator

252
Q

Aortic stenosis and valve replacements

A

Calcitic degenerative growth cause yet unknown but must be treated early to avoid complete vessel occlusion
Mostly good recovery and valve replacement
Valve replacements -
Tissue valves can be used, made from animal products like pigs, treated to ensure no antigens and no rejection, lasts around 5 years in young active people so will need repeat surgery and around 15 years in less active elderly people so more commonly used in elderly. Mechanical valve lasts lifetime but requires lifetime warfarin drugs to thin blood avoiding blood clots from valve. Problematic in pregnancy so try to avoid in young women wanting children but not always possible to avoid. Most common valve replacement in young people.

253
Q

Left and right coronary arteries

A

Left anterior descending artery supplies the anterior 2/3 septum and anterior wall of LV
circumflex artery supplies 15% AVN, 45% SAN, anteriolateral papillary muscle mitral valve

Right coronary artery supplies 55% sinus node, right ventricle, 85% atrioventricular node, posterior 1/3 septum if dominant.

Right or left can be dominant - good indicator is if one of the branches has more branches or is larger than the other or shows a posterior branching.

Involved in ischaemic heart disease and acute coronary syndrome, left ventricle aneurism etc.
Managed with medical therapy, coronary stents, coronary artery bypass surgery depending on each individual case.
Stents act as vessel wall scaffolding but increase risk of clot reoccurring therefore also contain anti-inflammatory drugs.
CABG - can use the internal mammary arteries some use both but no evidence patients do better with both than compared to great saphenous vein from leg and mammary artery works just as well. Can be done on or off pump.

254
Q

Which side are foreign substances more likely to fall down in the bronchi

A

Right side as is steeper

255
Q

Lung resections

A

Lung is divided into lobes and then further into segments each with their own functional units - pulmonary artery and vein and bronchi branch.
Following vascular supply means segments can be separated for tumour removal etc without removing an entire lobe or lung.
Segmentectomy - removal of small functional unit of lung
Lobectomy - whole lobe
Pneumonectomy - entire lung.
Type of resection determined by remaining salvageable lung, how much damage removal will do, where the object or tumour is and airway etc.
Some sections are too complex to remove alone so whole lobes or multiple sections are taken eg centre of lung more difficult than periphery.
Vascular supply to tissue directly from the aorta to supply lung tissue usually running along the fissures of the lung. Surgeons look for these during resectioning.

256
Q

Pulmonary embolism treatment

A

Rarely treated with surgery usually patients do just as well with anti-coagulant drugs

257
Q

Innervation of respiratory muscles and surgery

A

Intercostal nerves
Phrenic nerve from C3,4,5(keeps diaphragm alive)
Accessory muscles supplied by brachial plexus, cervical plexus, accessory nerves
Very small collateral branch on top of ribs also but in surgery these don’t need to be avoided as are very small.
Use rib spreader to mobilise muscles as well as bone to help reduce pain. Robotic arms are better at reducing pain due to the angle they can enter the thorax.

258
Q

Flail ribs

A

Fractures in more than once place along rib separating segment from rest of cage and segment moves in during inspiration and out in expirations opposite to normal rib movement. Can cause hyperinflation lungs extend more and more and can’t empty.
Fixing arms to allow accessory muscles to assist breathing.

259
Q

Pleural cavity abnormalities

A

Pneumothorax - space between visceral and parietal pleura filled with air compresses lung need to insert tube to drain chest using chest flap - one way valve seals during inspiration and drains in expiration.
Air can displace heart and other organs which can cause cardiac arrest so must be dealt with quickly
Visceral pleura become sticky with blood sticking to chest wall and preventing collapse again
Mesothelioma - cancer associated with asbestos exposure, layers of pleura get tumour and infiltrates lung preventing inflation of lung causing trapped lung.

260
Q

Posterolateral thoracotomy

A

Incision through intercostal space on back often spread with rib spreaders to mobilise muscles and reduce pain.
Scapula good landmark for indication of 6th rib position for incision. Cut lat dorsi and serrated anterior underneath use rib spreaders to avoid intercostal damage.
Robotic access can be less painful as angle of entry is form above not laterally.
Laparoscopy can be used in some cases of smaller surgery but large incisions are needed for larger removal due to severe consequences of not being able to reach internals fast enough if something went wrong.

261
Q

Mesenteric root

A

Origin - 15 Cm length extends from duodenjejunal flexure 2.5cm left of midline at L2 inferior to transverse mesocolon to right sacroiliac joint.
From right to left it crosses :
Third part of duodenum, aorta, IVC, right psoas major, right ureter, gonadal vessels and ilacus.

262
Q

Midgut and embryology

A

Begins caudal to liver bud level of D2
Extends to junction 2/3 along transverse colon
Supplied by SMA
Innervated by vague nerves sympathetic and parasympathetic from superior mesenteric plexus.
Primitive gut tube undergoes craniocaudal and lateral folding is suspended from body wall by dorsal mesentery. Rapid elongation of gut and mesentery gives primary intestinal loop connected at apex to yolk sac outside embryo by vitelline duct.
Intestinal rotation week 6 - Physiological umbilical herniation due to rapid elongation and lack of space 270° total. Elongation and coiling of small intestine and large bowel and caecum formation.
Week 10 - retraction of loops back into abdominal cavity led by proximal jejunum lastly by caecum. Further 180° rotation so transverse colon lies in front of duodenum.

263
Q

Omphalocoele

A

Herniation of viscera through enlarged umbilical ring
Failure of bowel to return after herniation
Contents covered by peritoneum and amnion

2.5 in 10,000 births high mortality 25% associated with severe malformations (cardiac 50%, neural tube defects 40%)

264
Q

Gastroschisis

A

Herniation of abdominal contents through abdominal wall directly into amniotic cavity
Lateral to umbilicus usually at site of regression of right umbilical vein - weak structure
No peritoneal covering

1 in 10,000 births no association with mortality or severe defects but volvulus bowel may result in ischaemia

265
Q

Meckels diverticulum

A

Blind ending pouches arising from anti mesenteric boarder 50-100 Cm variable size from iliocaecal valve 1-5 Cm length in 75%

In 2% of population mucosa mainly ileal can contain gastric, pancreatic, duodenal mucosa which may cause erosion and perfusion.

Freely mobile or attached to anterior wall at umbilicus may be tethered by cord or cystic remnants of Vitello-intestinal duct. Sometimes form fistula at umbilicus leaking contents externally.
If tethered can loop and twist around compromising blood supply

266
Q

Portal vein formation

A

SMV joins with splenic vein behind neck of pancreas in transpyloric plane to form portal vein into the liver

267
Q

Small bowel comparison jejunum and ileum

A

Jejunum - thicker, more vascular, more numerous larger thicker valvulae connections, larger diameter, thinner less fatty mesentery, 1 or two arcades with infrequent terminal branches and NO PEYERS PATCHES
Ileum - thinner, less vascular, fewer smaller thinner valvulae (circular folds), mesentery is thicker and more fat with 3-5 arcades and frequent terminal vessels, peyers patches present along antimesenteric boarder

268
Q

Lymphatics of small intestine

A

Nodes within mesentery near vasa recta
~200 nodes
Lymph flows along intestinal arteries to larger superior mesenteric lymph nodes at root of mesentery
Lymph carries nutrients from bowel plays role in regulating fluid homeostasis and immune surveillance

269
Q

Nerve supply to small intestine

A

Sympathetic from lower thoracic spinal cord to coeliac plexus via sympathetic trunks and splanchnic nerves - pain fibres
Parasympathetic from vagus CN X - increase motility, secretion and vasoconstriction
Both end on and modulate the activity of the enteric nervous system

270
Q

Intussusception

A

Proximal segment of intestine -intussusceptum - invaginate into adjacent distal segment - intussuscipiens
Meckels diverticulum is cause in children, unknown 85%
Other causes - polyps, duplications, atresias, lymphoid hyperplasia.
In adults 2/3 known cause, 2/3 tumours, 2/3 malignant.
In children treat with air reduction and resection as unlikely from tumour.
In adults resection needed as may be from malignant cause

271
Q

Malrotation

A

Results from variation from normal midgut rotation and fixation during development
Broad base of normal small bowel mesentery protects against rotation
Interrupted rotation results in duodenojejunal flexure lying on right and caecum on left
Narrow base of mesentery leaves small bowel prone to twisting resulting in midgut volvulus and necrosis of bowel
Duodenum doesn’t cross midline
Surgery needed - untwist bowel and fix caecum on left and SB on right as well as perform appendicectomy to avoid appendicitis later in life and it being missed or not diagnosed due to odd location

272
Q

Ileocaecal valve

A

Protrudes into caecum
Two rings of thickened circular muscle form closing mechanism
In large bowel obstruction a competent valve prevents dispersion of intralumenal pressure resulting in closed loop obstruction with risk of perforation

273
Q

Small bowel obstructions

A

Band adhesion - peritoneal bands from previous surgery can cause small bowel obstruction so again need surgery to resolve obstruction
Ileus - no mechanical obstruction but gas distributed throughout and bowel function disturbed, likely stomach distension.
Crohn’s disease - often present as terminal ileitis, inflammation and pain, stricturing causing obstruction, biopsies taken of terminal ileum at colonoscopy
Trauma - central abdominal distribution makes it vulnerable for perforations after trauma

274
Q

Macroscopic large bowel

A

Appendices epiploicae - fat filled peritoneal tags 2-5 Cm over surface most numerous in sigmoid not found on small bowel.
Taeniae coli - 3 flattened bands compromised of longitudinal muscle shorter than bowel creating sacculations run from base of appendix to recto sigmoid junction

275
Q

Marginal artery of Drummond

A

Runs close to mesenteric edge of colon provides communication between SMA and IMA and mobilisation of long lengths of bowel in resectional and reconstructive surgery in bowel problems.

276
Q

Lymphatic drainage and nerve innervation of large bowel

A

Superior midgut and inferior hindgut has mesenteric lymph nodes - lymph
Innervation - sympathetic fibres from lower thoracic region of spinal cord with arteries - pain aside from sigmoid where is from pelvic splanchnic nerves. Reduces muscular activity and vasoconstriction
Parasympathetic - differs for midgut and hindgut increases muscular activity and pain from sigmoid
Vagus to midgut
Pelvic splanchnic nerves s2-4 to hindgut

277
Q

Relations of appendix

A

Medial caecum below IC valve - McBurney’s point
Varying length and position
Retrocaecal most common 64% cases - overlies iliacus And psoas can cause spasm and right hip flexion with pain on hip extension
Pelvic 32% may be near right ureter, ovary Fallopian tube. Difficult to diagnose
Pre- ileal
Post ileal

278
Q

Relations of caecum

A
In right iliac fossa superior to lateral half of inguinal ligament
Posterior - ilacus, psoas, femoral nerve
Anterior - abdominal wall when distended
Lateral - ab wall
Medial - ileum
Inferior - base of appendix
279
Q

Hepatic flexure relations

A

Level of L3 surface marking - costal margin at 10th costal cartilage
Posterior - lower pole of right kidney
Anterior - right lobe of liver

280
Q

When mobilising splenic flexure be careful of

A

Tearing splenic capsule in hemicolectomy

281
Q

Splenic flexure watershed region

A

Splenic flexure at risk of iscaemia - marginal artery of Drummond provides communication between distal branches of SMA and IMA to colon
Inconstant alternative communicating artery between proximal SMA and IMA
In case of occluded IMA splenic flexure and descending colon rely on these communications for arterial supply

282
Q

Rectum

A

Begins at rectosigmoid junction S3
Ends at dentate line
Follows curve of sacrum and coccyx ending 2-3 Cm anterior to tip of coccyx
No teniae coli, appendices epiploicae or well defined mesentery
Blood supply - terminal branch IMA superior rectal artery, branch of pudendal artery supplies anal canal.
Veins go from rectum to superior reveal vein IMV then to portal vein
Lymph - upper and middle rectum has pararectal nodes then inferior mesenteric nodes
Variable region Low to middle rectum across ischiorectal fossa along internal pudendal vessels to internal iliac nodes
Below dentate line - anal canal - is superficial inguinal and external iliac nodes
Nerves - sympathetic from L1/2 via IM and hypogastric plexuses. Post synaptic nerves distributed in walls of superior rectal artery and it’s branches
Sympathetic constricts internal sphincter
Parasympathetic from pelvic splanchnic nerves s2-4 increase muscle activity but relax internal sphincter. Carry pain fibres.
Below pectinate line of anus nerve supply is somatic from inferior recital nerve - branch of internal pudendal s2-4

283
Q

Right hemicolectomy

A

Respect distal ileum, caecum, ascending colon and proximal to mid transverse colon
For caecal/ascending colon/hepatic flexure cancers
Ileocolic and right colic branches divided below origin. Marginal artery also divided and right branch of middle colic
Extended right hemicolectomy - resect right side of colon up to or beyond splenic flexure and attach ileum to remaining colon. Right colic and both branches of middle colic artery are divided
Ascending branch of left colic artery is preserved giving excellent blood supply to anastomoses
For tumours of hepatic flexure, transverse colon with it without splenic flexure
Mobility of terminal ileum allows tension free anastomoses to remaining colon
Transverse colectomy rarely performed as high anastomotic leak rates

284
Q

Left hemicolectomy

A

Resection of transverse colon left of middle colic vessels to level of upper rectum
Tumours of descending colon and sigmoid with/out splenic flexure
Radical resection requires division below origin of left colic artery
Division of left IMV improves mobility of transverse colon
Sigmoid colectomy preserves ascending branch of left colic

285
Q

Low anterior resection and TME

A

Rectal resection with anastomoses to remaining rectum - 5cm distal to tumour for upper rectal cancers
- 2cm distal to tumour for middle and low rectal cancers
Total mesorectal excision TME of complete fascial envelope surrounding the rectum (containing lymph nodes) to level of ani muscles must be performed in order to improved local control and reduce rates of recurrence

286
Q

Abdomino-perineal resection

A

Removal of rectum including the sphincter complex for low rectal cancers
Requires formation of permanent stoma
IMA divided below origin at aorta

287
Q

Oesophageal developmental abnormalities

A

Tracheoesophageal fistula or oesophageal atresia caused by failure of partition of oesophageal and trachea. Connections between tubes
Often seen with other birth defect problems 1 1/2 of all babies with TE fistula or OA have another birth defect.
Symptoms noted soon after birth include - bubbles in mouth, coughing or chocking, vomiting, blue colour of skin, breathing difficulty, full round abdomen.

288
Q

Oesophageal sphincters

A

Upper- lower part of inferior constrictor of pharynx, cricooharyngeus and upper part of circular muscle of oesophagus
Closed at rest - 100mmHg resting pressure
Protects airway from gastric content reflux
Lower - not demonstrated anatomically, physiological - has basal tone but relaxes for swallow

289
Q

Close relationship between oesophagus and trachea is important in surgery because ..

A

Pushing on trachea will close oesophagus and close lumen preventing backflow during surgery

290
Q

Indentations of oesophagus

A

Visible on barium swallow
Arch of aorta
Left main bronchus
Left atrium - not usually but if pathologies are present such as mitral stenosis resulting in enlarged atrium causes difficulty swallowing and obvious narrowing

291
Q

Foreign bodies and caustic burns in oesophagus

A

Three major constrictions
- cricopharyngeal at start of oesophagus at ~17cm from upper incisors
- bronchoaortic (aorta at T4 left main bronchus T5) at ~28cm
- diaphragm T10 ~43cm
75% of foreign bodies ingested lodge in oesophagus
Lower boarder of cricoid denotes start of oesophagus (narrowest point of gut except appendix) likeliest site of an obstruction getting stuck
If object swallowed passes through oesophagus it will likely be small enough to pass through entire GIT. If anything magnetic or corrosive is swallowed must be removed to prevent fistulas

292
Q

Arterial supply to oesophagus

A

Subclavian
Inferior thyroid in neck
Upper right intercostal arteries in superior mediastinum
Branches of thoracic aorta - direct oesophageal branches
Left gastric (coeliac axis) and inferior phrenic (abdominal aorta)

293
Q

Oesophageal weak points

A

Above and below pharyngeal muscle particularly around cricopharyngeal are possible site of pulsion diverticulae - can harbour squamous cell carcinomas

294
Q

Oesophageal Portal hypertension

A

Anastomoses between azygos (systemic) and left gastric (portal) tributaries in oesophageal veins
Most commonly arises secondary to liver cirrhosis
Increased hepatic vascular resistance redirects flow towards azygos through portosystemic anastomoses at lower oesophagus
Can cause oesophageal varices - enlarged distended oesophageal veins and risk rupture and potential catastrophic bleed.

295
Q

Oesophageal lymph and nerve supply

A

Lymph nodes everywhere very problematic in cancers spreading - use endoscopic biopsy to determine extent of spread and possibilities of surgery and treatment etc
Innervation - motor voluntary, recurrent laryngeal supplies striated muscle of upper oesophagus
Intrinsic nerve network (part of enteric nervous system) capable of causing coordinated muscle action in absence of extrinsic nerves
Parasympathetic from vagus and recurrent laryngeal nerves peristalsis is major motor effect
Sympathetic from cervical sympathetic trunk, vasoconstriction and pain

296
Q

Dysphagia

A

Intrinsic and extrinsic causes
Intrinsic - benign or malignant structures and tumours
Extrinsic - lymph nodes at root of lung, aortic aneurism, enlarged right atrium in mitral stenosis

297
Q

Acid reflux

A

Prevented by sphincter at cardiac orifice
Supplementing closure of orifice - angle of His (oesophagus entering stomach) - diaphragmatic muscle action (tenses in coughing etc to prevent reflux
Lower oesophageal oesophageal mucosal metaplasia switching squamous to columnar epithelium called Barrett’s oesophagus - increases risk of carcinoma needs to be endoscopically screened for cancers

298
Q

Hiatus hernia

A

Part of the stomach migrating up into the thorax
Very common over 50, doesn’t need treatment if it is asymptommatic
Can travel past the diaphragm cause is somewhat unknown but thought to be from damaged weakened muscle tissue or repeated pressure on muscles around the stomach.
Type one - most common, 90% cases stomach bulges up into thorax not folding just moved superiorly to normal position - feeds through crux of diaphragm
Type two - 3% cases stomach folds upwards next to oesophagus compressing both spaces
Type three - mixture of one and two, can cause stomach twisting in on itself causing surgical emergency. tend to need surgical repair to pull stomach inferiorly again and unfold other section back into anatomical position.
Type four - entire bowel starts to pass through diaphragm Crux can have at least 50% abdomen in thorax
Surgery - complete resection of hernia sac, reduction of herniated stomach and 2-3cm distal oesophagus into abdominal cavity to prevent reoccurrence and repair diaphragm defect.

299
Q

Stomach development

A

Rotates - left wall becomes anterior and right becomes posterior
Posterior walls grow faster and forms the greater curvature while the central wall forms the lesser curve
So well supplied by blood is highly unlikely to suffer ischaemia
Lymphatics also very extensive which is bad for cancers of the stomach and spread

300
Q

Gastric ulceration surgery

A

Mainly medical and endoscopic management and treatment surgery is very uncommon as if it is needed is a huge operation so try to avoid - vagotomy

301
Q

Epiploic foramen in surgery - Pringle manoeuvre

A

Site of potential hernia - bowel travelling through into lesser omentum
Also enables the Pringle manoeuvre - clamping the whole with fingers or weak tie cuts blood supply to vessels present in this mesentery which is good for surgery and a good anatomical landmark

302
Q

Pyloric stenosis

A

Obstruction - vomit that is non bilious
Hypertrophy of mainly circular muscle at pylorus
Extreme narrowing of pyloric lumen
Pylorus can be thickened and elongated
Need to cut muscle, correct hydration in patient before surgery or anaesthetic could kill them

303
Q

Duodenal abnormalities

A

Atresia or stenosis
Probably due to lack of recanalisation during development
Occasionally secondary to annular pancreas
Tumours in head of pancreas may cause compression

304
Q

Presentation of abdominal pain

A

Sympathetic afferents carry pain from foregut to thoracic cord
Pain from stomach and in duodenum to papilla experienced in epigastrium
Pain from common site of duodenal ulcer - duodenal cap - felt in epigastrium

305
Q

Define hernia

A

Abnormal protrusion of organs or tissue through the wall of cavity which it normally resides or from one body cavity to another
Symptoms include pain, physical limitation around the area, cosmetic problems and skin problems - eg thinning or perforating skin.
Looks like a lump on the skin. Can be easily reduced or irreducible, tender or not etc.
An irreducible tender hernia is a surgical emergency while an easily reducible hernia may not even need operation.

306
Q

Where Can hernias occur

A
Inguinal 
Femoral
Umbilical
Paraumbilical
Epigastric
Incisional
Spigelian
Obturator
307
Q

Hernia complication

A

Strangulation - protruding contents is restricted by hole and venous congestion occurs from insufficient drainage followed by arterial ischaemia
Incarceration - irreducible hernia that is not obstructed
Obstruction - bowel obstruction (distension, abdominal pain, vomiting, absolute constipation)

308
Q

Hernia surgery

A

Routine operation
Aim to reduce hernia and effectively repair defect
Small repair uses stitches while larger repairs may need mesh also
Elective repair is best - open or laparoscopic
Emergency surgery has higher risk of infection and may need bowel resection
Risks dependent on patient risk factors in particular obesity and condition of patient is very important. More obese have higher likelihood of reoccurrence.
May develop mesh infection which is very complicated needs to be removed from area ASAP and allowed to clear infection.

309
Q

Abdominal wall layers

A
Skin
Campers fascia
Scarpas fascia
External oblique
Internal oblique
Transverse abdominis
Transversalis fascia
Extraperitoneal fat
Peritoneum
310
Q

Arcuate line

A

Horizontal line demarcates lower limit of posterior rectus sheath

311
Q

Umbilical/paraumbilical hernia and epigastric hernia

A

Umbilical ring closes after ligation of the umbilical cord can take time and so hernias in babies are common but mostly settle with time
They are acquired in adults due to weakness of abdominal wall
Increased risk factors are pregnancy and obesity or excessive stomach straining eg weight training.
Make incision above or below umbilicus and determine where healthy tissue is - remove dead tissue and excess fat.
Either push hernia back into abdomen or remove it
Suture individual layers
Skin around umbilicus is often thin and so contents may be stuck to walls making finding planes very difficult increasing risk of bleeding or bowel injury.
Mesh placed ideally pre-peritoneal but if not possible then only specific types must be used as increases risk of bowel injury
Epigastric hernia - defect in linea alba treated the same as umbilical.

312
Q

Divarification of the recti

A

A weakness of the linea alba - not a hernia and thus can’t be repaired and not much can be operated on due to surgical risk of bowel injury.

313
Q

Laparotomy and hernia

A

Commonly midline incision for access to abdomen in bowel surgery
Nowadays is very small incision
Is a risk for incisional hernia - hernia occurring at site of incision due to wound strength only reaching max 80% of original tissue weakens wall increasing hernia risk
Can occur due to poor surgical closure
Or patient factors like obesity, poor nutrition and infection etc.

314
Q

Mesh types for hernia repair

A

Superficial to deep
Onlay - under abdominal fat just below skin is at risk if injury is sustained of infection
Inlay - peritoneum layer
Retromuscular - posterior to muscles
Preperitoneal - anterior to peritoneum ideal mesh placement
Intraperitoneal - posterior to peritoneum directly on bowel not ideal need specific mesh types only increased infection risk

315
Q

Inguinal canal anatomy

A

Anterior - external oblique aponeurosis
Posterior - conjoint tendon medically and transversalsis fascia laterally
Superficial ring immediately above and medial to pubic tubercle

316
Q

Inguinal hernia

A

More common in males due to developmental descent of testes causing weakness in abdominal wall
Direct hernia from wall weakness is medial to epigastric vessels
Indirect when contents passes through deep ring is lateral to epigastric vessels ends up in scrotum or labia.
Sometimes difficult to clinically determine and may both co-exist in same patient.
Clinically - put finger on deep ring and ask patient to cough is hernia doesn’t come out it’s likely indirect if it does it’s likely direct
Repair - Lichtenstein open hernia repair is most common repair used. Incorporates mesh and tension free repair. Need to identify hernia, separate hernial sac, reduce it and place mesh.

317
Q

Femoral hernia and compare to inguinal hernia

A

Hernia into femoral canal typically affects older women
More likely to lead to small bowel obstruction than inguinal hernia
Femoral hernia more common in women but inguinal hernias are overall more common
Inguinal hernia is above and medial to pubic tubercle and femoral is below and lateral to pubic tubercle
Femoral hernia is more likely to have strangulation because of rigid neck Ricters hernia and surgery is a must because of strangulation while inguinal can also be treated without surgery.

318
Q

Femoral triangle boarders

A
Superior - inguinal ligament
Lateral - sartorious
Medial - adductor longus
Contents - nerve, artery, vein, empty space (canal) and lymphatics 
NAVEL
319
Q

Spigelian hernia

A

Defect in linea sumilunaris - aponeurosis if internal oblique, external oblique and transversus abdominis

320
Q

Liver

A

Largest organ in body
2.5% body weight approx 1500g
Occupies all right hypochondrium and epigastrium extends into left hypochondrium inferior to the diaphragm
Covered by peritoneum aside from bare area, porta hepatis and bed of gall bladder.
Coronary ligament - Reflection of peritoneum from diaphragm to liver, has anterior and posterior layers which meet to form the right triangular ligament. Posterior layer continuous with lesser omentum
Left triangular ligament - formed by falciform ligament and lesser omentum
Falciform ligament - peritoneal reflections from upper anterior abdominal wall from umbilicus to liver. Has ligamentum teres I’m free edge
Blood supply - dual blood supply from common hepatic 30% right and left and aberrant arteries - accessory right from SMA 7%, replaced right SMA 11%, accessory left from left gastric 8%, replaced left from LG10%
Portal vein 70% drains from entire GIT.
Porta hepatis via hepatoduodenal ligament.
Venous drainage directly to IVC via right, middle and left hepatic veins.

321
Q

Liver recesses and spaces

A

Subphrenic Recess - superior extension of PC between diaphragm and superior aspect of liver. Separated into left and right by falciform ligament
Subheptic space - immediately inferior to liver
Hepatorenal recess - Morrison’s pouch posterior extension of subhepatic space between right part of visceral surface of liver, right kidney and suprarenal gland. Space is gravity dependent. Communicates anteriorly with right subphrenic recess.
Can get abscesses and fluid here need to be drained - use ultrasound to guide the needle, also used in liver biopsy. Enter via 10th IC space mid axillary line during expiration. Drain can be placed into any cavity for abscess

322
Q

Bile duct

A

Intrahepatic - in the portal triad divided as per segmental anatomy
Extrahepatic - common hepatic divides right and left, common bile duct below cystic duct variable length 5-15cm descends posterior to superior part of duodenum opens into ampulla with pancreatic duct.
Blood supply - cystic artery supplies proximal duct, right hepatic artery to middle, posterior superior pancreatoduodenal and GDA to retroduodenal duct.
Two main arteries run at right and left boarder of bile duct - variable arise from retroportal, retroduodenal or GDA and communicate with rift hepatic artery or sometimes left HA.

323
Q

Pancreas

A

Retroperitoneal posterior to stomach
Subdivided into head, neck, body, tail
Head embraced by C shape of duodenum, uncinate process projects from inferior part of head. Rests posteriorly on the IVC right renal artery and vein and left renal vein. Bile duct lies in groove on posterosuperior surface or embedded.
Neck is short and overlies superior mesenteric vessels and SMV joins splenic vein to form portal vein.
Body anterior surface covered in peritoneum and floor of lesser omentum. Posterior surface has no peritoneum and contacts aorta, SMA, left suprarenal gland left kidney and renal vessels
Tail anterior to left kidney related to splenic hilum very mobile region with higher abundance of islet cells
Blood supply - branches of splenic artery form arcades with GDA and SMA branches. Head supplied by anterior and posterior superior pancreatoduodenal arteries from GDA and AP inferior PD from SMA. Venous drains mainly into splenic vein

324
Q

Pancreatic ducts

A

Exocrine function - pancreatic juice enters duodenum via ducts
Main duct begins at tail and runs through to head turns inferiorly and closely related to bile duct. In most cases main duct and bile duct unite forming ampulla of Vater in D2 at major duodenal papilla.
Accessory pancreatic duct - santorini. Opens into minor duodenal papilla 60% communicate with the main PD.

325
Q

Pancreatic resection

A

Head - due to intimate relationship with duodenum needs to also resect duodenum and bile duct. Needs reconstruction - hepaticojejenostomy, pancreaticojejenostomy, gastojejenostomy.
Tail - usually also needs splenectomy
Drainage procedures - drain cysts directly into stomach

326
Q

Functional division of the liver

A

Divided into eight not four lobes based on vessels not surface anatomy and ligaments
Right and left divided by portal triad
Each lobe divided vertically by hepatic veins
Divided horizontally by subdivision of portal triad vessels
Important for liver resections

327
Q

Liver resection

A

Mainly needed for cancers usually secondary to bowel cancers as due to portal vein most likely to secondarily spread to the liver from bowel.
Use intravenous planes to help resection
Anatomical resection - left and right hemihepatectomy, left and right trisectionectomy, left lateral sectionectomy and caudate lobectomy.
Non-anatomical section - wedge resection with clear restriction margin for removal of small benign tumours etc.
Type of removal depends on blood supply, amount of remaining healthy tissue and amount of tissue needing to be removed. Taking too much might cause liver failure.
Often a biliary reconstruction is needed due to removal of the gall bladder as well, need to perform a hepaticojejunostomy creating false bile duct by dividing jejunum 15-20cm downstream of ligament of Trez and anastomoses to the common hepatic duct.

328
Q

Liver transplantation

A

Classic transplant - donor organ placed with vessels into recipient and recipient organ removed
Piggyback transplant - donor organ placed on top of recipient liver
MeVis scans used to map out livers blood supply and lobes in great detail to determine if organ is transplantable

329
Q

Whipple procedure and distal pancreatomy

A

Aka a pancreaticoduodenectomy - complex operation removing the head of the pancreas and first part of duodenum, gallbladder and bile duct.
Reconstruction needed after for bile duct.

A distal pancreatectomy is a surgery to remove a tumor from the body or tail of your pancreas

330
Q

Gallbladder surgery and cholecystectomy

A

Very common due to gall stones being very common likely to cause problems with infection, pain and occlusion.
7-10cm long lies in gallbladder fossa on liver surface
Subdivided into fundus, body which contracts and neck directed toward porta hepatis.
Hartmanns pouch- our pouching of gallbladder wall at junction of the neck and cystic duct.
Calots triangle aka cystohepatic triangle - right and left hepatic arteries and cystic artery travel through here - important landmark for surgery. Forms critical view window necessary for finding correct vessel to clamp in removing the gallbladder.
Divide cystic artery and duct and dissect GB off liver however prone to odd anatomy making this difficult. Can use intraoperative cholangiogram inject dye into cystic duct to trace correct vessel and ensure correct one is removed.

331
Q

Define Cystogastrostomy

A

Cystogastrostomy is a surgery to create an opening between a pancreatic pseudocyst and the stomach when the cyst is in a suitable position to be drained into the stomach. This conserves pancreatic juices that would otherwise be lost

332
Q

Obstructive jaundice

A

jaundice resulting from blockage of the bile ducts or abnormal retention of bile in the liver.
Recommended the gallbladder is removed within two weeks of onset
Once removed use duct to collect remaining stones using balloon or basket
Can use angiogram to check vessel obstruction
Endoscopic retrograde cholangiopancreatography ERCP easiest surgical method to remove stones - endoscope uses identify duodenal papilla, cannulate and inject dye then extract stones.
Very close to pancreas likely to cause irritation 1-5% risk pancreatitis
PCT route needle through liver into bile duct to drain above the level of the obstruction is access can’t occur through ERCP.